You are on page 1of 200

LISTENING

GIỚI THIỆU:

“ …TOEIC (viết tắt của Test of English for International Communication – Bài
kiểm tra tiếng Anh giao tiếp quốc tế) là một bài thi nhằm đánh giá trình độ sử dụng
tiếng Anh dành cho những người sử dụng tiếng Anh như một ngoại ngữ (không
phải tiếng mẹ đẻ), đặc biệt là những đối tượng muốn sử dụng tiếng Anh trong môi
trường giao tiếp và làm việc quốc tế. Kết quả của bài thi TOEIC phản ánh mức độ
thành thạo khi giao tiếp bằng tiếng Anh trong các hoạt động như kinh doanh,
thương mại, du lịch… Kết quả này có hiệu lực trong vòng 02 năm và được công
nhận tại nhiều quốc gia trong đó có Việt Nam.

Trong lớp học này:


+ Tiếp cận toàn bộ kiến thức từ các tài liệu mới nhất
+ Ôn tập theo dạng đề sát nhất như tại IIG
+ Bài tập được thiết kế theo ngày để học viên được duy trì luyện tập.
+ Số điểm tối đa cho khung chương trình trên là 550 điểm

Thông qua lớp học này, các em có thể đạt được kỹ năng nghe hiểu cơ bản, kỹ năng
nghe tóm tắt và nghe hiểu chuyên sâu. Đạt tối thiếu 60/100 LC và 45/100 RC trên
phầm điểm của bài thi TOEIC. Làm nên móng giúp đỡ các học viên vững bước
trên con đường chinh phục tiếng Anh, nâng cao kỹ năng học tập và làm việc …

Unienglishcenter.

UNIENGLISHCENTER 1
LESSON 1&2 – PART 1
I – EXAMINATION: Focus on the picture and select the best describe listed
bellow
1 1.
A) He’s picking up a bag
B) He’s cycling on a road
C) He’s climbing some rocks
D) He’s wearing a jacket

2 2.
A) A man is pushing a shopping cart
B) A man is paying for some groceries
C) Some merchandise is arranged on
shelves
D) Some baskets are lined up on the floor

3 3.
A) Items have been placed in a shopping
cart
B) Some vegetables are on display in a
store
C) Ceiling lights are being installed
D) Flowers are being planted in pots

UNIENGLISHCENTER 2
Sơ đồ phân tích tranh

Dạng 1: Bức tranh chỉ có 1 người:

Giới thiệu:

- Đáp án thường cho biết hành động của nhân vật trong bức ảnh

- 4 đáp án đều có dạng: S + is + V-ing + …. và có cùng chủ ngữ, động từ


đóng vai trò quyết định đáp án đúng – sai.

- Trong trường hợp, đáp án đúng không nói đến hành động của nhân vật trong
bức ảnh mà cho biết trang phục hoặc diện mạo của nhân vật.

- Các đáp án sai thường đề cập đến sự vật nổi bật trong bức ảnh

Những điều cần lưu ý:

- Xác định hành động của nhân vật trong bức ảnh

- Tập trung nghe các động từ xuất hiện trong các đáp án. Nếu chỉ tập trung
nghe danh từ mà không nghe được động từ thì bạn rất dễ chọn sai đáp án.

- Nếu đáp án có động từ và danh từ không diễn tả hành động và sự vật trong
bức ảnh thì đó là đáp án sai

UNIENGLISHCENTER 3
II – LANGUAGE BULDING: Listen to and repeat!
-
FOOT ACTION
Be sitting around the fountain Ngồi xung quanh đài phun nước
Be standing behind the counter Đứng sau quầy hàng
Be walking up a stairway Bước lên cầu thang
Be strolling along the water’s edge Đi dạo dọc bờ sông
Be crossing a street Băng qua đường
Be waiting by the train tracks Đợi ở cạnh đường ray
Be leaning on the railing Tựa vào lan can
Be riding a bicycle by the water Đạp xe đạp bên bờ sông
Be boarding a bus Lên xe buýt
Be getting into the vehicle Bước vào xe
Be exiting through a door Đi ra ngoài cửa
Be disembarking from the aircraft Xuống máy bay

HAND ACTION
Be holding up some merchandise Cầm một vài món hàng lên
Be holding headphones with both hands Cầm headphone bằng cả hai tay
Be lifting a chair off the floor Nhấc ghế lên khỏi sàn nhà
Be reaching for a book Với lấy một quyển sách
Be pushing a button ấn nút
Be pushing a cart Đẩy xe đẩy hàng
Be pointing at something on a paper Chỉ vào cái gì đó trên tờ báo
Be positioning a sheet of paper on the Đặt một tờ giấy lên mặt kính
glass
Be placing a file folder Đặt một tập tài liệu
Be taking a book from the bookcase Lấy một cuốn sách từ tủ sách
Be turning a page Lật một trang giấy
Be throwing away some garbage Vứt rác
Be unfolding a map Mở một tấm bản đồ
Be carrying some plants Mang một vài cây

UNIENGLISHCENTER 4
EYES ACTION
Be looking at merchandise on display Ngắm nhìn hàng hóa được trưng bày

Be looking in a drawer Nhìn vào ngăn kéo

Be looking out the window Nhìn ra ngoài cửa sổ

Be examining some papers Kiểm tra một số tài liệu

Be facing some equipment Nhìn một số thiết bị

Be gazing out at the scenery Chăm chú nhìn bức tranh treo trên tường

Be focusing on the computer screen Tập trung vào màn hình máy tính

Be watching a misucal performance Xem buổi biểu diễn âm nhạc

Be viewing a document in a notebook Xem tài liệu trong vở ghi chép

FOOD
Be cooking some food Nấu vài món ăn

Be chopping up some vegetables Thái rau

Be wearing an apron Đeo tạp dề

Be setting a table Dọn bàn

Be using a sink Sử dụng bồn rửa bát

Be shopping for groceries Mua hàng tạp hóa

Be eating inside a restaurant Ăn tại nhà hàng

Be having a meal Dùng bữa

Be ordering food Gọi món

Be serving food to the customers Mang thức ăn cho khách

Be pouring water into a glass Rót nước vào ly

Be packing some food into the plastic Đóng gói thực phẩm trong hộp nhựa
container

UNIENGLISHCENTER 5
No. 1
- Checking - Pouring
- Examining - Pulling
- Inspecting - Pushing
- Looking at - Stacking
- Hearing into - Sweeping
- Reviewing - Operating
- Hanging up - Wiping
- Holding - Climbing
- Pointing at - Lying on
- Reaching for - Leaning against
- Taking an order - Writing on
- Paying for - Sitting at
- Adjusting - Standing in line
- Lifting - Waiting in line
- Loading - Boarding
- Mopping - Crossing
- Mowing - Exiting
- Carrying - Facing
- Cleaning - Waving
- Filling - Walking along
- Fixing - Talking on the telephone
- Repairing - Trying on
- Putting on

UNIENGLISHCENTER 6
No. 2
- Drawer - Merchandise
- Mechanical part - Musical instrument
- Microscope - Vehicle
- Notepad - Arch way
- Photocopier - Cash register
- Projector - Ceiling
- Rack - Construction site
- Wheel - Dock
- Work vest - Fence
- Beverage - Doorway
- Document - Hallway
- Equipment - Outdoor market
- Fabric - Pier
- Grocery - Pillow
- Item - Walkway
No. 3
- Be arranged - Be reflected
- Be attacked to - Be under construction
- Be being loaded - Be crowded with
- Be being unloaded - Be located
- Be hanging - Be docked
- Be harvested - Be stationed
- Be lined up in rows - Be covered with
- Be on display - Be sat
- Be displayed - Be laid down

UNIENGLISHCENTER 7
- Be packed - Above the bridge
- Be placed - Across the water
- Be positioned - Against the wall
- Be prop against - Along the lake
- Be scattered - Behind the vehicle
- Be secured to - Between the trees
- Be fastened to - Both sides of
- Be stacked up - From a ceiling
- Be piled up - Near the river
- Be stocked with - Next to the mirror
- Be filled with - On the platform
- Be suspended - Toward the stream
- Be tied to - In the corner of the room
- Be unoccupied - Under a counter
- Be emptied
- Be eliminated by

Dạng 2: Bức tranh có 2 người trở lên:

Giới thiệu:

- Đáp án đúng có thể mô tả trạng thái hoặc hành động của những người trong
bức ảnh nhưng cũng có thể tập trung vào một người. Do đó, chủ ngữ trong
đáp án đúng thường là các từ/cụm từ như they, the people, some people, a
man/ woman, one man/ woman ….
- Theo sau chủ ngữ trong các đáp án là động từ diễn tả hành động của chủ ngữ
và các sự vật nổi bật có liên quan
- Tình huống trong bức ảnh có thể xẩy ra ở nhiều địa điểm khác nhau như cửa
hàng, nhà hàng, đường phố,…
Lưu ý:

UNIENGLISHCENTER 8
- Xác định hành động của tất cả các nhân vật trong bức ảnh, chú ý đến nhân
vật nổi bật nhất.
- Lưu ý bẫy được sử dụng trong đáp án sai: chủ ngữ kết hợp với động từ diễn
tả hành động của nhân vật khác hoặc chủ ngữ là tất cả các nhân vật trong
ảnh kết hợp với động từ diễn tả hành động của một nhân vật.
- Đáp án có thể là câu bị động ở thì hiện tại tiếp diễn (chủ ngữ + is/are + being
+ p.p.) với chủ ngữ là sự vật chịu tác đông của hành động mà con người
đang thực hiện

Dạng 3: Mô tả bức ảnh có người và sự vật

Giới thiệu:

- Các đáp án có thể cho biết mối quan hệ giữa con người và sự vật nhưng
cũng có thể chỉ tập trung vào con người hoặc sự vật
- Chủ ngữ của các đáp án có thể là sự vật và động từ diễn tả hành động mà
con người đang thực hiện được dùng ở thể bị động của thì hiện tại tiếp diễn
(is/are + being +p.p.)
- Đáp án nói về khung cảnh hoặc sự vật không nổi bật trong bức ảnh cũng có
thể là đáp án đúng.
Lưu ý:

- Chú ý lắng nghe động từ khi bạn thấy nhân vật trong bức ảnh cầm đồ vật và
đang thực hiện hành động nào đó!
- Dù nhân vật trong bức ảnh đang thực hiện hành động thì trong nhiều trường
hợp, chủ ngữ của đáp án đúng không phải là nhân vật đó mà là sự vật và
động từ được dùng ở thể bị động của thì hiện tại tiếp diễn (is/are + being+
p.p.)
- Ngoài khung cảnh hay sự vật nổi bật trong bức ảnh, bạn cũng nên chú ý đến
các chi tiết nhỏ như đường phố hay bóng người.
- Hãy lắng nghe từng đáp án đồng thời quan sát bức ảnh và xác định thật
nhanh đáp án đó đúng hay sai.

UNIENGLISHCENTER 9
A – ACTUAL TEST
1 2

3 4

5 6

UNIENGLISHCENTER 10
B –TACTIC PRATICE
1 2

3 4

UNIENGLISHCENTER 11
C – MINI TEST
1 2

3 4

5 5

UNIENGLISHCENTER 12
III – POWER LISTENING: Fill out blanks and select the best describe!
ACTUAL TEST 1
1
A. She’s …………….. the door
B. She’s ………. ………. the sofa
C. She’s ………. a piece of ……….
D. She’s ………… a letter

2
A. There’s a ……… in the tower
B. There’s a towel ……… …………
C. There’s a …………. on the lobby
D. There’s a …….. next to the ………

3 A. He’s …………… a magazine


B. He’s ………. …….. a display case
C. He’s ………… ……… a vehicle
D. He’s ………… ………… a sign

UNIENGLISHCENTER 13
4 A. They’re ……… ……… at a desk
B. They’re ……….. each other
………. a computer
C. They’re …………… decorations
D. They’re …………… some
furniture in room

5 A. He’s ………….. a car


B. He’s …………….. a wheelbarrow
C. He’s …………. air ………. a tire
D. He’s …………… boxes

6 A. They’re ……… …….. a path


B. They’re ………….. the grass
C. They’re ……… ……… their maps
D. They’re ……….. in a circle

UNIENGLISHCENTER 14
7 A. He’s …………… bookshelves
B. He’s …….. ………… in boxes
C. He’s …………. a volume from the
shelf
D. He’s ………….. a book in paper

8 A. Chairs ……… ………. ………..


in the corner of the room
B. The silverware is being …………
C. Water is being …………. into the
glasses
D. The table ……. ……… …… for a
meal

9 A. He’s ………….. the show


B. He’s ……… …….. a sled
C. He’s ……… …….. his jacket
D. He’s …………. a hat

UNIENGLISHCENTER 15
10 A. A ship is ……… ……… the bridge
B. Water is ……… ………. of a pipe
C. The bridge has many ………… on
it
D. Some people are ………… ………
the bridge

ACTUAL TEST 2
1 A. They’re ……… ………. the
screen
B. They’re …. …….. …….. each
other
C. They’re ………. the desk
D. They’re ………….. the walls

2 A. She’s …………… some shoe boxes


B. She’s …………… a shoe
C. She’s ………. ………… a shelf
D. She’s ………….. a handbag

UNIENGLISHCENTER 16
3 A. He’s ………….. a tool belt
B. He’s ……….. ……….. a house
C. He’s …………… some wood
D. He’s ………. ………… this hat

4 A. Trees are being ……………..


down the river
B. Buildings are ………………. the
water
C. Boats are ……….. ………… the
bridge
D. Children are …………… in the
fields

5 A. He’s …….. ………. a machine


B. He’s ……….. ………. his sleeves
C. He’s ………… an electrical cord
D. He’s ………….. his jacket

UNIENGLISHCENTER 17
6 A. The women are ……………… a
desk
B. A woman is …………. in a
notebook
C. Some books are ………. ………
on the floor
D. A customer is ………….. for some
merchandise

7 A. A ………………… is being made


B. They’re …………….. the walls
C. One man is ………… the back of
a chair
D. Cabinets are being ……………..

8 A. She’s ……………… for a book


B. She’s ……………… a machine
C. She’s …………. the pages of a
newspaper
D. She’s …………. ………… some
wallpaper

UNIENGLISHCENTER 18
9 A. The driver is ……………. the
engine
B. Some tires are being
……………….
C. Some workers are …………. boxes
D. The truck is …………. next to
containers

10 A. The men are ………… work vests


B. The men are …………….. a shelf
C. The men are …………… traffic
D. The men are ……………. outdoors

ACTUAL TEST 3
1
A. He’s …………….. a briefcase
B. He’s ……………….. a hat
C. He’s ………………. a plane
D. He’s ………………… the ground

UNIENGLISHCENTER 19
2 A. The man is ……….. ……… a
sheet of paper
B. The man is …………… a drawer
open
C. The man is ……………. gifts
D. The man is ………………..
shelves

3 A. She’s …………….. a box


B. She’s …………….. a cart
C. She’s ………….. a floor
D. She’s …………….. a button

4 A. They’re ……………. in a shop


B. They’re ……………. some steps
C. They’re …………….. outdoors
D. They’re …………. bricks

UNIENGLISHCENTER 20
5 A. The people are …………… the
pies
B. The desserts are ……….
…………….
C. The prices are …………..
………….
D. The pies are being …………..

6 A. A man is ……………. a picture


B. A man is ……….. ………. a
purchase
C. A man is ………. ……... a
photograph
D. A man is …………… a camera

7 A. The man is ………….. his watch


B. The bicycle is ………. ……… the
ground
C. The man is ……………… the shop
D. The bicycle is in a ………….
………

UNIENGLISHCENTER 21
8 A. The woman’s ………… are in her
…………
B. The woman is ………………. in
the garden
C. The woman’s …….. is on the
………
D. The woman is ………………. by
flowers

9 A. He’s ……………. his luggage


B. He’s ………. …………. his
briefcase
C. He’s ……….. ……….. a laptop
computer
D. He’s ……………. two computers

10 A. The woman is ………….. a cup of


coffee
B. The woman is ………… …………
the machine
C. The woman is …………….. up a
document
D. The woman is ………………….
the machine

UNIENGLISHCENTER 22
ACTUAL TEST 4
1 A. She’s ……………. her hands
B. She’s ……………. gloves
C. She’s …………….. her hair
D. She’s …………… ……….. her
coat

2 A. They’re ……….. ……….. cups


B. They’re ………….. a discussion
C. They’re ………… ………… their
glasses
D. They’re …………. in a conference
room

3 A. A man is …………. a paper


B. A woman is ……….. notes
C. Some people are being ……….. to
their seats
D. Some people are ……….
…………. the window

UNIENGLISHCENTER 23
4 A. They’re ……… …………
notebooks
B. They’re ………….. for their plates
C. They’re ……… …….. …….. one
another
D. They’re ………… food and drinks

5 A. The …………. are in the …………


B. The airplanes are ………… at a
high altitude
C. The airplanes are ……. ……….
D. The airplanes are ……….. on the
ground

6 A. He’s ………….. into a bookcase


B. He’s ………. some furniture
C. He’s …………… a wire
D. He’s …………. a hole in a piece of
wood

UNIENGLISHCENTER 24
7 A. The man and woman are
………….. their luggage
B. The woman is ………. on the board
C. The woman is …………… a book
to the man
D. The man has a bag …….. ……
…………….

8 A. She’s ……… ……….. curtains


B. She’s ……. ……….. a hat
C. She’s ………. ……….. a cup
D. She’s ………… ………. a picture

9 A. He’s ………… baggage down the


steps
B. He’s …………… suitcases in both
hands
C. He’s ……….. suitcases from a
vehicle
D. He’s ……………. next to a
baggage trolley

UNIENGLISHCENTER 25
10 A. The chairs are ……… ……….. the
wall
B. The audience is ……. ………
C. The chairs have been ………… in
rows
D. The presentation has ………….

ACTUAL TEST 5
1 A. They’re …………. some
photographs
B. They’re …………… a meeting
C. They’re ………… their watches
D. They’re ………….. …………….

2 A. The …………. has been


………….. by the window
B. The beds have been ……………
together
C. There’s a ………… above each
………
D. There’s a …………. …………….
the beds

UNIENGLISHCENTER 26
3 A. The woman is ………… …………
a painting
B. The woman is ……………… art in
a …………….
C. The woman is …………….. a
picture
D. The woman and man are
…………….. some artwork

4 A. The crates are …………….


B. The vegetables are ……………….
C. The goods are ……… ……………
D. The bottles have been ……………..

5 A. The boat is ……. ……. ………….


B. The boat is ……….. ……… at the
dock
C. Some boats are …………… each
other on the water
D. There are many ……… next to the
………….

UNIENGLISHCENTER 27
6 A. He’s …………. a tie
B. He’s …………. the chairs
C. He’s ……………… the table
D. He’s ………………. a document

7 A. The office is ………. …………


supplies
B. Someone is ……….. ……… the
computer
C. Supplies are being ………………
in the warehouse
D. The shelves in the office are
……………

8 A. They’re ……………. the ladder


B. One man is …………… a can of
paint
C. They’re ……………. onto the roof
D. One man is ……………. a long
pole

UNIENGLISHCENTER 28
9 A. The fountain is ……………….
water into the air
B. People are …………….. in the
pool
C. Water is ………….. …………..
the mountain
D. Children are ……………… from
the water fountain

10 A. The woman is ………… …………


the cabinet
B. The ………. to the clothes closet is
…………
C. The workers are ………. …………
their equipment
D. The lab …………….. is locking the
door

UNIENGLISHCENTER 29
NOTEPAD:

UNIENGLISHCENTER 30
LESSON 3 – PART 2: WHO – WHEN – WHERE
– WHAT/WHICH – WHY.
A – WHO
I – EXAMINATION: Read the text carefully and select the best answer!
1. Who responsible for preparing breakfast this 2. Who sent you such a gorgeous dress?
morning?
A. My mother did
A. In the living room
B. You shouldn’t have said so
B. Before you get to work
C. For her birthday
C. It’s your turn

3. Who was the man at the bar last night? 4. Whose shopping bag is this on the table?
A. Just a drink or two A. Oh, you decided to go shopping
B. That was a client from England B. Yes, this is the table we reserved
C. It’s my favorite bar C. That belongs to me

II – LANGUAGE BUILDING: Listen to and repeat!


- Receptionist - The board of directors
- Plumber - Vice president
- Accountant - CEO
- Consultant - Human resources department
- Mechanic - Accounting
- Technician - Sale department
- Architect - Marketing department
- Assistant - Customer service department
- Supervisor - Shipping department
- Project manager - Maintenance department
- Director

UNIENGLISHCENTER 31
III – TEST TACTIC
A. Listen to the questions and possible responses, then choose the best response to each question.
1. (A) (B) (C) 5. (A) (B) (C)
2. (A) (B) (C) 6. (A) (B) (C)
3. (A) (B) (C) 7. (A) (B) (C)
4. (A) (B) (C) 8. (A) (B) (C)
B. Listen again and fill in the missing words to complete the gaps.
1. Who …………… the sales ……………? 5. Who is ……………………. the matter?
A. I …………… it was Ms. Gomez A. I asked Jane to ……………………. it.
B. No, I …………… to Mr. Johnson B. It’s actually ……………………...
C. ………………………… C. I’m ………………………….. as well
2. Who is …………………..the new 6. Who has a ………. of the ………….….?
……………?
A. …………………….. at least.
A. John’s been …………… to it.
B. I’ll get us …………………..
B. I’ve looked …………… for it.
C. Check with …………………………..
C. The new ……………….. sits here
7. Who’s giving the …………………….. at
3. Who is ……………….come and pick up today’s ………………….?
the ……………?
A. Sandy from the ………………………
A. I’ll ………………………… later
B. He will be …………………. today
B. John will be here ……………
C. Yes, ………………………… lunch
C. Let’s ……………… over once again
8. Who is the new ……………………….?
4. Who gave you the ………………….?
A. Her ………………… is Brenda
A. Mr. Ashford did
B. In the reception ………………….
B. She had many …………………
C. No, it is not new
C. I did not …………….. it to him.

UNIENGLISHCENTER 32
IV – MINI TEST
A. Listen to the questions and possible responses, then choose the best response to each question.
1. (A) (B) (C) 5. (A) (B) (C)
2. (A) (B) (C) 6. (A) (B) (C)
3. (A) (B) (C) 7. (A) (B) (C)
4. (A) (B) (C) 8. (A) (B) (C)
B. Listen again and fill in the missing words to complete the gaps.

1. Who ……………………… the 5. Who is John with now?


……………………. we get?
A. ……………………………………..
A. Ms. Rodriguez, I think
B. In ……………………………….
B. We get them ……………………
C. Let’s go and see him
C. Yes, it is a …………………….
6. Who are you ……………………………?
2. Who’s going to get the ……………….
for the ………………? A. I’ve looked ……………………….

A. At the …………………………… B. A red ………………………. I left

B. No, not ………………... C. I’m here to see Mr. Singh

C. I’ll do it 7. Who ……………………. all the money


in your ………………………….?
3. Who asked for a copy of the …………?
A. My wife does
A. Yes, I will ask him
B. At the …………………………….
B. ………………………………..
C. ……………………..dollar bills
C. Mr. Johnson did
8. Who got here first?
4. Who ………………… the new schedule?
A. Mr. Kwan did
A. You will have to ………………. it
B. Let’s ………………………. soon
B. Sorry, I can’t change …………………...
C. I got it as well
C. I think Mr. Sutton did.

UNIENGLISHCENTER 33
B – WHEN
I – EXAMINATION
1. When will the job training session begin? 2. When did you return from America?
A. During the first week of June A. Next June
B. The party room is upstairs B. Yes. I have an exam this afternoon
C. Yes. It will be held here in San Francisco C. Last December

3. When is your assignment due? 4. When did you move to this city?
A. About a couple of pages long A. It’s too boring
B. Next month B. More than three months ago
C. On your survey C. Most weekend

II – LANGUAGE BUILDING
- Yesterday - Almost everyday
- Last year - On a weekly basis
- Three weeks ago - Sometimes
- A couple of days ago - Quite recently
- Since last spring - This afternoon
- In the past - In 10 minutes
- The day before yesterday - Not until next Monday
- These days - By the end of the week
- Right now - Later today
- Currently - Within a week
- Sometime next month

UNIENGLISHCENTER 34
III – TEST TACTIC
A. Listen to the questions and possible responses, then choose the best response to each question.
1. (A) (B) (C) 5. (A) (B) (C)
2. (A) (B) (C) 6. (A) (B) (C)
3. (A) (B) (C) 7. (A) (B) (C)
4. (A) (B) (C) 8. (A) (B) (C)
B. Listen again and fill in the missing words to complete the gaps.
1. When do you want to …………………? 5. When’s the best time to …………… you?
A. ……………………………. A. Try me …………………………..
B. Sure, take it B. Let’s sign the ……………………
C. No, I don’t want it C. The best is yet to come
2. When will Ms.Chow …………… from 6. When will the …………….. from Japan
her ………………………… to Shanghai? arrive?
A. She speaks Chinese A. ……………………….
B. …………………………………. B. ……………………….
C. To land a contract C. ……………………….
3. When is the …………………… due? 7. When is Dr. Jones available for an
………………………?
A. ………………………….
A. This ………………………. at 2 is open
B. To ask for a raise
B. Yes, I will be ……………………. then
C. Due to ………………………….
C. He is at the hospital today
4. When’s your ………………………?
8. When will the annual conference
A. I am very ……………………… ………………………….?
B. …………………………………. A. ……………………………….
C. He’s not here B. I’ll meet you there
C. ………………………………..

UNIENGLISHCENTER 35
IV – MINI TEST
A. Listen to the questions and possible responses, then choose the best response to each question.
1. (A) (B) (C) 5. (A) (B) (C)
2. (A) (B) (C) 6. (A) (B) (C)
3. (A) (B) (C) 7. (A) (B) (C)
4. (A) (B) (C) 8. (A) (B) (C)
B. Listen again and fill in the missing words to complete the gaps.
1. When will the ……………………… be 5. When is the report ……………..?
completed?
A. …………………………….
A. Yes, I’ll do it …………………….
B. To the ……………………..
B. In ……………………………
C. I’ll do it later
C. I’ll assign it later
6. When does the …………….. meeting
2. When is the …………………. form due? ………………………….?
A. For the ………………….. position A. The place is not too far
B. I will apply for it, too B. ………………………….……………..
C. By the 31st of …………………….. C. In the ……………………………. room
3. When will the ……………….. be ready? 7. When is the merger ………………….. to
go through?
A. Please be ready soon
A. By …………………… of next year
B. To give it out at the …………………
B. No, I didn’t expect it
C. …………………………….. at 2
C. To get to the other side
4. Do you know when the online ……….….
……………. will open to the public? 8. When are we going to get audited?
A. No, it’s a private matter A. Yes, it will be edited
B. Probably in ………………….. B. To get it …………………………
C. Yes, I know that places well C. ………………………………………

UNIENGLISHCENTER 36
C – WHERE
I – EXAMINATION
1. Where did you go for your honeymoon? 2. Where should I put the recharged pen?
A. In the middle of winter A. Put it on the desk
B. Thailand for 7 days B. Whenever you have spare time
C. With my mother-in-law C. You sure can do it by yourself

3. Where has Ms. Garcia gone? 4. Where will you be staying while you’re in
London?
A. At three o’clock
A. With an old friend
B. To see Mr. Jones
B. Not at this time of year
C. No, not yet
C. It was a dinner invitation

II – LANGUAGE BUILDING
- In the conference room - Right over there
- On the third floor - To the right
- In the auditorium - To the left
- Down the street - In the filing cabinet
- At the west terminal - Somewhere in the north
- On the next corner - Warehouse
- Down stairs - Auditorium
- Up stairs - Head quarters
- Right across the hall - Main office
- Behind the building - Art exhibition
- In front of the store - Conference
- Press conference

UNIENGLISHCENTER 37
III – TEST TACTIC
A. Listen to the questions and possible responses, then choose the best response to each question.
1. (A) (B) (C) 5. (A) (B) (C)
2. (A) (B) (C) 6. (A) (B) (C)
3. (A) (B) (C) 7. (A) (B) (C)
4. (A) (B) (C) 8. (A) (B) (C)
B. Listen again and fill in the missing words to complete the gaps.
1. Where did Mary go? 5. Where is the …………………………….?
A. ………………………………………. A. To manage it
B. She is doing great B. Generally not
C. I talked to her C. …………………………………….
2. Where can I ………………. John Smith? 6. Where do you …………………?
A. He’s on the ………………………… up A. Let’s leave soon
B. I found it in the ………………….. room B. In this …………………………
C. He was ………………………….. you C. …………………………………
3. Where did Ms. Curtis ………………. the 7. Where would you like me to …………?
weekly report?
A. ……………………………… your name
A. She left ……………………….. ago
B. Wait for the signal
B. On top of …………………………
C. On the …………………………., please
C. No, it’s a …………………………
8. Where is the …………………………..?
4. Where will the ……………… seminar be
held? A. To make a withdrawal

A. …………………………. B. Next to the …………………………….

B. …………………………. C. Let’s go there together

C. For all the employees

UNIENGLISHCENTER 38
IV – MINI TEST
A. Listen to the questions and possible responses, then choose the best response to each question.
1. (A) (B) (C) 5. (A) (B) (C)
2. (A) (B) (C) 6. (A) (B) (C)
3. (A) (B) (C) 7. (A) (B) (C)
4. (A) (B) (C) 8. (A) (B) (C)
B. Listen again and fill in the missing words to complete the gaps.
1. Where do we keep the weekly 5. Where is the men’s ……………………..?
………………….?
A. The apartment ………………………….
A. Probably next ……………………
B. ………………………………………….
B. Let’s keep waiting a bit more
C. Only men are allowed
C. Try looking in the ………………………
6. Where did you ever find this old chair?
2. Where does the bus for Toronto leave?
A. …………………………………………
A. From gate …………..
B. Please have a seat
B. Yes, let’s go
C. Liz was ……………………………… it
C. ……………………..
7. Where is the hall for the awards
3. Where is the …………………….. room? ……………………….?
A. It’s on the ……………………………. A. …………………………………………
B. Let’s meet in ………………………… B. …………………………………………
C. We have plenty of room. C. For the best employee
4. Where do I have to go to get my 8. Where did Ms. Yamamoto put the
……………………………………………? ……………………………………………..?
A. ………………………….. at window 7 A. In the …………………………………...
B. You shouldn’t drive today B. She left an hour ago
C. I will go with you in a short while C. It’s over by now

UNIENGLISHCENTER 39
D – WHAT/WHICH
I - EXAMINATION
1. What kind of movie is showing at the Art 2. What can I get you for dink?
Cinema this weekend?
A. I’m sorry, I still don’t get it
A. It’s an excellent exhibit
B. There’s nothing that can be done
B. This weekend is really busy
C. I guess water would be good
C. I think it’s a foreign film

3. Which color should I use to highlight this 4. What do you think ò the movie last night?
point?
A. He went to the concert
A. Yes, its highlighted very nicely
B. I’ll see you at the meeting later
B. It’s been pointed out to me before
C. It was really exciting
C. Why don’t you use red?

III – TEST TACTIC


A. Listen to the questions and possible responses, then choose the best response to each question.
1. (A) (B) (C)
2. (A) (B) (C)
3. (A) (B) (C)
4. (A) (B) (C)
B. Listen again and fill in the missing words to complete the gaps.
1. ……………… does the restaurant close? 3. Which of these ………………. is yours?
A. ………………………………….. A. At the …………………………….
B. It’s close to the …………………. B. The black one is mine
C. No, it does not C. Thanks, it is
2. What do you think of this month’s ……? Which …………... developed this software?
A. Yes, …………………………… A. No, it’s too difficult
B. ……………………… we need money B. I’ll look it up
C. Thanks, it was a gift C. Quite recently

UNIENGLISHCENTER 40
IV – MINI TEST
A. Listen to the questions and possible responses, then choose the best response to each question.
1. (A) (B) (C) 7. (A) (B) (C)
2. (A) (B) (C) 8. (A) (B) (C)
3. (A) (B) (C) 9. (A) (B) (C)
4. (A) (B) (C) 10. (A) (B) (C)
5. (A) (B) (C) 11. (A) (B) (C)
6. (A) (B) (C) 12. (A) (B) (C)
B. Listen again and fill in the missing words to complete the gaps.
1. Which ……………………. should I buy? 6. Which …………………… should we eat
A. ………………………………..…….. dinner at?
B. The gray one. A. ………………… is at seven P.M.
C. In the dresser. B. ……………………. one is fine.
2. What’s the topic of today’s …………….? C. We can meet …………………….
A. In an hour. 7. What’s the hourly pay rate?
B. ………………………………………. A. No, it’s my old watch.
C. No, on the bottom. B. It’s theirs, not ours.
3. ……………………. does the train arrive? C. It’s ……………………………..
A. …………………………………………. 8. Which …………………… is yours, Jim?
B. Yes, I think it might rain. A. The one ………………………...
C. No, I won’t have time. B. From nine to five.
4. What do you think of this month’s C. It’s official.
budget? 9. What would you like to ……………..?
A. Yes, this month. A. Yes, thank you.
B. Looks like we need money. B. No, I don’t
C. Thanks, it was a gift. C. Orange juice, please.
5. What time are you ……………………...? 10. Which is the key to the ………………..?
A. Down at the harbor. A. You could file a complaint.
B. From ……… A.M. to ……….. P.M. B. The one with the red tag.
C. Yes, I have time. C. Fill it in later.

UNIENGLISHCENTER 41
11. What’s the …………………………… 12. What is the …………………….. of your
for this weekend? visit to Singapore sir?
A. No, there were five of them. A. Yes, five or six times.
B. They’re ………………………. rain. B. At a hotel …………………………….
C. It’s a week from now. C. I’m here for a ………………………...

E – WHY
I – EXAMINATION
1. Why did Jacson call the hotel? 2. Why are you here so late?
A. It’s in the lobby A. That’s the wrong date
B. To cancel our reservation B. I have to finish the budget
C. I’ll call everyone else C. No, we haven’t lately

3. Why didn’t you ride your bicycle to work? 4. Why is the shopping center closed?
A. It’s supposed to rain A. The sale lasts all week
B. From a bike shop downtown B. It’s being renovated
C. He didn’t write to me C. Mainly clothes and shoes

II – LANGUAGE BUILDING
- Because of a schedule change - To meet with the customer

- Because of wrong size - To thank us for our hard work

- Because I will out of town - In order to get a refund

- Because he was tired - In order to finish early

- Due to heavy traffic - For a business trip

- Due to severe weather - For a dentist appointment

UNIENGLISHCENTER 42
III – TEST TACTIC
A. Listen to the questions and possible responses, then choose the best response to each question.
1. A) B) C)
2. A) B) C)
3. A) B) C)
4. A) B) C)
B. Listen again and fill in the missing words to complete the gaps.
1. Why are the……………………. here? 3. Why was the ………………… changed?
A. They should be here soon A. Because Mr. Wang couldn’t be here then
B. No, at the escalator B. I will schedule an ………………………
C. To ………………. a new computer lab C. Sorry, I don’t have any
2. Why isn’t the …………….. working? 4. Why did you ………... so late last night?
A. It’s ………………………………… A. In 1 hour
B. Print ………………………….. here B. ………………………………………….
C. A lot of work C. Until next week

IV – MINI TEST
A. Listen to the questions and possible responses, then choose the best response to each question.
1. A) B) C) 6. A) B) C)
2. A) B) C) 7. A) B) C)
3. A) B) C) 8. A) B) C)
4. A) B) C) 9. A) B) C)
5. A) B) C) 10. A) B) C)

UNIENGLISHCENTER 43
B. Listen again and fill in the missing words to complete the gaps.
1 - Why has the traffic stopped moving? 6 - Why don’t you ……….. us for dinner
A) Maybe there’s some tonight?
……………………………….. A) No, they haven’t yet.
B) Yes. I’d love to see a movie. B) Thanks, but I have other plans.
C) No thanks, I can take the train. C) To attach some new parts.
2 - Why’s the store ………... so early today? 7 - Why don’t you ………………………….
A) No, it isn’t very close. with us?
B) Yes, I got up at six today. A) I don’t eat peaches.
C) It’s a ………………………………. B) Yes, he arrives today.
3 - Why is Mr. Suzuki …………….. to C) Sure. …………………………………...?
South America? 8 - Why was the …………………………..?
A) I didn’t know you were moving. A) Because Mr. Wayne couldn’t be here
B) She’s …………………... to a new office. then.
C) The plane arrives at nine. B) I’ll …………………….. an appointment.
4 - Why did you ………... so late last night? C) Sorry, I don’t have any.
A) In one hour. 9 - Why are there so many …………. in the
B) We had a long meeting. …………….. today?
C) Until next week. A) It needs a lighter road.
5 - Why did Mrs. Chang ………….. early? B) There’s a special event later.
A) She had a doctor ……………………... C) I’ll drive myself, thanks.
B) About thirty minutes ago. 10 - …………………………. take a few
C) She’s usually ………………….. days off and get some rest?
A) …………………………………...
B) No, now’s not a ………………. time.
. C) The others are cheaper.

UNIENGLISHCENTER 44
D – ETS ACTUAL TEST 1 E – ETS ACTUAL TEST 2

NOTEPAD

UNIENGLISHCENTER 45
LESSON 4 – PART 2: HOW + CAN/COULD
A – HOW
I – EXAMINATION
1. How long has it been since I saw you last? 2. How do you usually go to the shopping mall?
A. I think this dress is too long A. By bus
B. I used to sew quite a bit B. Not often
C. Quite a few years C. On Saturday

3. How many résumé have you received so 4. How do I get to the accounting office?
far?
A. They have a new manager
A. It’s not far
B. Take the stairs to the third floor
B. In the newspaper
C. Can you help me count these?
C. There have been several

II – LANGUAGE BUILDING
- In writing - By weekly
- In person - For 2 days
- By bus - Once in a while
- By plane - More than 10 years
- By credit card - At least once a month
- By overnight delivery - Within the next month
- Through fundraising event - Usually just on Saturdays
- Through an internet search

UNIENGLISHCENTER 46
III – TEST TACTIC
A. Listen to the questions and possible responses, then choose the best response to each question.
1. (A) (B) (C)
2. (A) (B) (C)
3. (A) (B) (C)
4. (A) (B) (C)
B. Listen again and fill in the missing words to complete the gaps.
1. …………............. is this 3. How ……………………… to the
………………….……? restaurant tonight?
A. It’s going to ……………….. A. I will take the bus
B. ……………………… euros B. No later than …………………….
C. It won’t take long C. I think I will ……………………..
2. How ………………………….. your 4. How ……………………………. at the
new job? hotel?
A. Yes, they are hiring now A. …………………………., I can’t
B. I went to ……………………………. B. Near Ford ………………………
C. About …………………………. ago C. I enjoyed it

IV – MINI TEST
A. Listen to the questions and possible responses, then choose the best response to each question.
1. (A) (B) (C) 6. (A) (B) (C)
2. (A) (B) (C) 7. (A) (B) (C)
3. (A) (B) (C) 8. (A) (B) (C)
4. (A) (B) (C) 9. (A) (B) (C)
5. (A) (B) (C) 10. (A) (B) (C)

UNIENGLISHCENTER 47
B. Listen again and fill in the missing words to complete the gaps.
1 - How …………………… to the 6 - How ……………….. can you park here?
restaurant tonight? A. I walk there every day.
A. I’ll take the bus. B. Only for an hour.
B. No later than seven o’clock. C. In the park.
C. I think I’ll rest a while. 7 - How ………………………………….
2 - How ………… will it be before we get this position at Norman Press?
to the ……………..? A. I read about it in the newspaper.
A. It’s three meters along. B. I’m still learning how.
B. About 15 minutes or so. C. It opens at 9:00 A.M.
C. No, not by bus. 8 - How …………….. is the meeting room?
3 - How ………….. is the gymnasium open? A. Big enough for thirty people.
A. I’m not late. B. On the tenth floor.
B. It closes at eight. C. Yes, it should be.
C. No, that’s OK. 9 - How ………………. is it going to cost?
4 - How ……………………………………. A. Yes, it was.
Jennifer’s retirement? B. $75.
A. More than twenty years. C. By cash.
B. Let’s throw her a party. 10 - How ……………. is the hotel from the
C. She’s very tired. train station?
5 - How ……………….. does a museum A. About ten minutes ago.
……………………. cost? B. The hotel is fairly small.
A. It’s ten euros. C. Only a few minutes away.
B. It’s not far.
C. By one o’clock.

UNIENGLISHCENTER 48
B – CAN/COULD/WOULD/MAY
I – EXAMINATION
1. Would you like more iced tea? 3. Could you tell me how often the bus leaves
for Madrid?
A. Yes, I would
A. There’s one every hour
B. They’re very nice
B. Only two pieces, please
C. I see it, too
C. No, she’s the trainer
2. Could you lend me your dictionary?
4. Can you play tennis this weekend, or are you
A. It’s over there on the shelf too busy?
B. No, I couldn’t send the letter A. I’d love to, but I don’t have time
C. He’s leaning on the desk B. I’m pleased to be here
C. The park has courts, though

II – TEST TACTIC
A. Listen to the questions and possible responses, then choose the best response to each question.
1. (A) (B) (C) 3. (A) (B) (C)
2. (A) (B) (C) 4. (A) (B) (C)
B. Listen again and fill in the missing words to complete the gaps.
1. May I ……………… the meeting early? 3. Why don’t we have pizza for dinner?
A. ………………………………………… A. Yes, it is peaceful tonight
B. I am meeting him next Tuesday B. ……………………………………..
C. ………………………………………… C. ……………………………………..
2. ……………………………… at the new 4. Would you like to …………………..
catalog? this weekend?
A. Yes, I knew that A. ……………………………………....
B. Because I bought some ……………… B. ………………………………………
C. Yes, …………………………………. C. I met him at 9 o’clock

UNIENGLISHCENTER 49
III – MINI TEST
A. Listen to the questions and possible responses, then choose the best response to each question.
1. (A) (B) (C) 5. (A) (B) (C)
2. (A) (B) (C) 6. (A) (B) (C)
3. (A) (B) (C) 7. (A) (B) (C)
4. (A) (B) (C) 8. (A) (B) (C)
B. Listen again and fill in the missing words to complete the gaps.
1 - Would you like to address our annual
convention next September? 5 – Could you order these supplies today?
A. Let me check my schedule. A. I was surprised, too
B. Here’s his extension. B. I’ll do it right away
C. It’s a user’s manual. C. In the cabinet
2 - Can you send me a copy of the revised 6 - May I ask you a question?
contract? A. No, I never have.
A. Yes, it’s a compact car. B. Last June.
B. He can contact us. C. Yes, how can I help you?
C. Yes, I’ll mail it today. 7 - Would you like me to fill out this form
3 - Would you like to go hiking this for you?
weekend? A. Yes, please do.
A. That would be great. B. No, it’s warm enough.
B. About twice a month. C. It’s a full pot.
C. I met him at nine o’clock. 8 - Would you like another cup of coffee?
4 - Can you give me change for ten dollars? A. It’s one dollar per cup.
A. Sure, do you want coins? B. I’ve been coughing all day.
B. It’s changed a little. C. Thank you, it’s delicious
C. It often breaks down.
.

UNIENGLISHCENTER 50
D – ETS ACTUAL TEST 3 E – ETS ACTUAL TEST 4

NOTEPAD

UNIENGLISHCENTER 51
LESSON 5 – PART 2: YES/NO QUESTION & /OR/
A – YES/NO QUESTIONS
I – EXAMINATION
1. Have you already booked a flight ticket? 2. The dental clinic is open on Sunday, isn’t it?
A. No, I was too busy A. The dentist was excellent
B. Yes, all the ticket was sold out B. Yes, until 2 P.M
C. My suitcases was full C. The doctor’s office is closed now

3. Was Mr. Cox at the meeting on Monday? 4. Did Jane go to the bank to deposit the checks?
A. He is the company president A. There is one near City Hall
B. No, he had another appointment B. Yes, she did that this morning
C. Yes, he will be there C. The bank closes at 4 o’clock.

II – LANGUAGE BUILDING
No.1

- Nobody knows - I am still considering it


- I have no idea - I am still waiting
- I am not sure - I am still thinking about it
- I am not certain - The manager is reviewing it
- We are not sure yet - I haven’t decided yet
- Not that I know of - It hasn’t been decided yet
- I don’t know anything about it - It depends on the design
- I wish I knew

No.2

- I prefer a window seat - I like both of them


- I feel like eating out - It doesn’t matter
- I will take a bigger one - It is up to you

UNIENGLISHCENTER 52
- It is nicer outside - Whichever you like
- Let’s stay indoors - I don’t like either of them
- I’d better go soon - I prefer neither

No.3

- Certainly - That’s what we expected


- Absolutely - I think so too
- Definitely - I am sorry, but…
- Why not - Unfortunately
- No problem - Sorry, I have an appointment then
- Not at all - Actually, I already have
- I’d love to - I am almost done, thanks
- I’d be happy to - I’ll try, but it is not going to be easy
- I am glad to - I am afraid I can’t
- I’ll be delighted to

III – TEST TACTIC


A. Listen to the questions and possible responses, then choose the best response to each question
1. (A) (B) (C) 6. (A) (B) (C)
2. (A) (B) (C) 7. (A) (B) (C)
3. (A) (B) (C) 8. (A) (B) (C)
4. (A) (B) (C) 9. (A) (B) (C)
5. (A) (B) (C) 10. (A) (B) (C)

UNIENGLISHCENTER 53
B. Listen again and fill in the missing words to complete the gaps.
1. He’s in the ……………………………..., 6. Her plane is two hours late, isn’t it?
…………………….....isn’t he? A. ……………………………………, yes.
A. OK, I’ll tell him “no”. B. Two hours ago.
B. ………………………………………... C. ………………………………………….
C. No, in the ……………………………... 7. Are you …………………………………
2. Isn’t there a ……………………… to the early today?
airport? A. Yes, at three o’clock.
A. It’s a ……………………………. ticket. B. The leaves are brown.
B. Yes, I cut my hair …………………….. C. I left it on your desk.
C. No, this is the …………………………. 8. That’s a ………………………, isn’t it?
3. …………………………………., isn’t it? A. Of course you can.
A. Yes, could you B. Yes, I bought it yesterday.
…………………..……………………? C. I didn’t see the news.
B. No, I didn’t call you. 9. Isn’t Mr. Williams ……………………?
C. Yes, I could do that. A. Yes, he should be here now.
4. That’s not today’s ………………………. B. Yes, it is ……………………………….
, is it? C. …………………………………………
A. Yes, I bought it this morning. 10. Is Ms. Burns likely to
B. …………………………………………. …………………………………………….?
C. OK. I won’t. A. Probably not.
5. Isn’t the address ………………………… B. Yes, I like it a lot.
be at the top of the page? C. ………………………………………….
A. ………………………………………….
B. It doesn’t have to be.
C. ………………………………………….

UNIENGLISHCENTER 54
IV – MINI TEST
A. Listen to the questions and possible responses, then choose the best response to each question
1. (A) (B) (C) 5. (A) (B) (C)
2. (A) (B) (C) 6. (A) (B) (C)
3. (A) (B) (C) 7. (A) (B) (C)
4. (A) (B) (C) 8. (A) (B) (C)
B. Listen again and fill in the missing words to complete the gaps.

11. Did you …………………………… last


weekend? 15. Do you know ………………………..
A. That one comes first. the bus comes?
B. No, I didn’t ……………………………. A. No, he doesn’t come here often.
C. They’re quite …………………………... B. The box came in this morning.
12. Jane, …………………………………… C. About every …………………………
with anyone? 16. Don’t you think we should
A. Thank you for that advice. ……………………………………………?
B. I asked my assistant to finish it. A. OK, but just …………………………..
C. Yes, with Cindy from the B. He can fix it.
…………………………………… C. It’s pretty tall.
13. Mr. Thompson likes ………………….., 17. ………………………………………….
doesn’t he? reading the report?
A. It’s his ……………………. A. Just two more pages.
B. An Italian car. B. He reports to Mr. Garcia.
C. No, I don’t own any. C. Yes, the apartment is furnished.
14. Didn’t you …………………………… 18. Rita …………………….. our
last night? newspaper subscription, didn’t she?
A. I’ll mail it in the morning. A. Yes, I have it delivered.
B. There’s the music store. B. No, she’s going to do it tomorrow.
C. Yes, I went with a friend. C. You should take it with meals.

UNIENGLISHCENTER 55
III – /OR/_TEST TACTIC
A. Listen to the questions and possible responses, then choose the best response to each question.
1. (A) (B) (C) 6. (A) (B) (C)
2. (A) (B) (C) 7. (A) (B) (C)
3. (A) (B) (C) 8. (A) (B) (C)
4. (A) (B) (C) 9. (A) (B) (C)
5. (A) (B) (C) 10. (A) (B) (C)
B. Listen again and fill in the missing words to complete the gaps.
1. Is the new furniture arriving today or 6. Did you receive the papers I faxed to you,
tomorrow? or should I fax them again?
A. I like driving to work. A. I haven’t seen them so far.
B. Yes, all the furniture is new. B. Yes, I should throw it out.
C. Neither. It’s going to arrive next week. C. Probably by train.
2. Is Ms. Lee here today or is she still in 7. Do you need to leave immediately, or can
Japan? you stay a little longer?
A. Yes, she’s from Japan. A. The sleeves are too long.
B. To visit her family. B. No thanks, I don’t need one.
C. She’ll return on Wednesday. C. I’d better go soon.
3. Are you going to watch the movie or the 8. Do you want to sit in the cafeteria or on
game? the patio outside?
A. Neither; I’m too tired. A. This is very good coffee.
B. Yes, I’ll move it here. B. Let’s stay indoors.
C. It’s not my watch. C. Pizza and a soda, please.
4. Do you want to have lunch delivered or 9. Do you want me to take the highway or
go to a restaurant today? Parker Avenue?
A. I love pizza, too. A. Won’t the highway be faster?
B. Let’s order in, since it’s raining. B. Let’s get out of the hallway.
C. I’ll do it myself. C. No, I don’t want one, thanks.
5. Do you want my home or work phone 10. Do you want to wait, or would you
number? rather come back later?
A. She leaves work at five. A. I don’t mind waiting.
B. It’s 52 Broad Street. B. Just one kilogram, please.
C. Whichever one I can reach you at. C. The one in the back, please.

UNIENGLISHCENTER 56
D – ETS ACTUAL TEST 5 E – ETS ACTUAL TEST 6

NOTEPAD

UNIENGLISHCENTER 57
LESSON 6 – PART 3.1 – CÂU HỎI VỀ VIỆC LÀM CỦA NHÂN VẬT
I – EXAMINATION
1. What are the speakers
discussing?
W: Did you hear that Joe Cooper is
A. A contest going to be interviewed for the evening
news?
B. An advertisement
M. Yes. It’s so exciting to have an
C. An interview engineer from our company on TV! Do
D. A concert you know when it is?

2. When will the broadcast take W. I heard they’re interviewing him on


place? Tuesday, but it won’t be broadcast
until Thursday.
A. On Tuesday
M. Thanks. I’ll have to remember to
B. On Thursday watch it when I get home that night.
C. On Friday
D. On Sunday
3. What does the man plan to do?
A. Purchase a watch
B. Have a television repaired
C. Schedule an interview
D. Watch a program

UNIENGLISHCENTER 58
II - LANGUAGE BUILDING
- Head quarter - Agenda
- Branch office - Hand-out
- Paper work - Chart
- Extension - Annual budget report
- Return one’s call - Market survey
- Security bag - Available
- Bulletin board - Scheduling conflict
- Transfer - Attend the meeting
- Identification card - Meet the deadline
- Promote sale - Postpone
- Sign a contract - Reschedule an appointment
- Submit a proposal - Reach an agreement
- Medical leave - Ahead of schedule
- Maternity leave - Behind schedule
- Access card - Come up with
- Flexible working hour - Office supplies
- Take a day off - Copier
- Get out of work - Computer components
- Call in sick - Installation
- Cover the shift - Stock room
- Work overtime - Malfunction
- Work late - Run out of
- Presentation - Place an order
- Meeting room - Out of order
- Conference call - Find a new supplier
- Video conferencing - Find a new vendor

UNIENGLISHCENTER 59
III – EXAMPLE

1.41. What are the speakers mainly discussing? 1.47. Who most likely is the man?
(A) A recent vacation (A) A mobile phone company employee
(B) An art class (B) A post office employee
(C) Plans for the weekend (C) A bank manager
(D) Their work schedules (D) A computer programmer

1.48. Why did the man call the woman? 1.50. What are the speakers discussing?
(A) To advertise a store opening (A) A play
(B) To ask about unpaid bills (B) A dance performance
(C) To ask for a form (C) A film
(D) To offer a special discount (D) A concert

1.56. What does the woman suggest going? 1.59. Where most likely does the woman
(A) To a restaurant work?
(B) To a coffee shop (A) At an employment agency
(C) To an ice cream shop (B) At a travel agency
(D) To a company cafeteria (C) At a property rental agency
(D) At an advertising agency

1.60. Where did the man read the 1.62. Why did the man go to the woman’s
advertisment? office?
(A) On a bulletin board (A) To discuss a report
(B) On a Web site (B) To ask for help
(C) In a brochure (C) To schedule a meeting
(D) In a newspaper (D) To request vacation time

2.41. Where do this conversation probably take 2.44. Where does this conversation take
place? place?
(A) In a restaurant (A) In a office
(B) In a hotel (B) In an apartment
(C) In an report (C) In a clothing store
(D) In a supermarket (D) In a furniture store

UNIENGLISHCENTER 60
IV – POWER LISTENING

1. What are the speakers (W-Am)` Hi Kyoko, it’s Ann, ― What


discussing? are you doing ______ _________? I
thought we might ____ __________ on
A. A recent vacation _________.
B. An art class
(W-Br) Sounds like fun, Ann. ― But my
C. Plans for the weekend _________ is visiting from ____ _____
________, and I’m _______ _____ to
D. Their work schedules
the _____ ________ on Saturday.
2. Who is visiting Kyoko?
(W-Am) Oh, I know your sister will
A. Her sister really ______ the art museum, ―I was
B. Her daughter just there ______ ______. The new
sculpture garden is really worth seeing.
C. Her mother
D. Her aunt
3. What did Ann do last week?
A. She visited a museum
B. She worked overtime
C. She saw a play
D. She went biking
4. What does the woman ask for? (W-Br) Daniel, can you give me
______ ______ after work today? My
A. A ride home car’s still in the ______.
B. A bus schedule (M-Br) ______, but I’m leaving work
early today, I have a ______ ______
C. An umbrella this afternoon at three.
(W-Br) Oh. Well, in that case, I can take
D. Advice about a car
______ ______. It’s only a ______
5. Where does the man need to go ______ to the bus stop from here, right?
at three o’clock? (M-Br) Yes, it’s not far. And, plus it
looks like the rain ______ ______ and
A. To the bus stop ______ ______is finally out.
B. To the train station

UNIENGLISHCENTER 61
C. To the auto shop
D. To the dentist’s office
6. What does the man say about
the weather?
A. It is cold
B. It is raining
C. It is windy
D. It is sunny
7. Where does this conversation W-Am) Good morning. This is the
probably take place? ______ ______. May I help you?
A. In a restaurant (M-Br) Yes, this is James Wagner in
B. In a hotel room ______ ______. I ordered ______
for eight o’clock this morning, but it
C. In an airport hasn’t ______ yet.
D. In a supermaket
(W-Am) I’m sorry Mr.Wagner. You
8. Why did the man contact the should have ______ it by now. I’ll call
woman? the ______ and check on it for you.
A. To inquire about an order (M-Br) OK, but i don’t ______ ______
B. To request a receipt ______. I need to be on the 9:30 bus to
______ ______ or I’ll miss my ______.
C. To buy a ticket
D. To make a reservation
9. Why is the man in hurry?
A. A restaurant is about to close
B. He is late for a business lunch
C. He has to catch a plane
D. A bus tour will begin soon

UNIENGLISHCENTER 62
10.Where does this conversation (W-Br) Excuse me. I’m ______
probably take place? ______a large bookshelf for my
______.
A. In a office
B. In an apartment (M-Am) What kind of _____________
are you looking for?
C. In a clothing store
(W-Br) I’d prefer a wall-mounted unit-
D. In a furniture store
but I’m most __________ about the
11.What is the woman most __________. I don’t want to spend more
concerned about ? than about $50.
A. The cost of an item (M-Am) We don’t have all our
B. The size of an office __________ on display. Let me go get
catalog for you so you’ll have a wider
C. The design of a machine __________ to choose from
D. The selection of items
12.What will the man probably do
next?
A. Sell a desk
B. Get a catalog
C. Select a model
D. Write an order
13.How long is the man’s (W-Am) Hi, Bill. Are you __________
presentation expect to last? for the __________ this afternoon?
You’ve got a two-hour __________ to
A. Half an hour give, right?
B. One hour
(M-Br) Right. But there’s a small
C. Two hours __________. My slides are finished, but
the __________ is broken and I can’t
D. Three hours
print out the __________ I want to
14.What problem does the man __________ at the meeting.
mention ?
(W-Am) Why don’t you call Jane and
A. The printer is broken see if she can print it for you on the
B. Some slides are missing __________ __________

UNIENGLISHCENTER 63
C. The meeting room is small
D. His presentation has been
postpone
15.What does the woman suggest?
A. Making an extra copies
B. Asking a coworker for help
C. Beginning a presentation early
D. Changing a meet room
16.What is the woman’s problem? (W-Br) Is there any way I can get on the
__________ __________ to Los
A. She lost the luggage Angeles? My flight from New York
B. She missed a fight didn’t arrive _____ __________so I
missed my connection.
C. She forgot her airplane ticket
(M-Am) Let’s see, I don’t have any
D. She doesn’t know where the gate
direct flights, but if you’re willing to
is
__________ in San Francisco, there is a
17.Where is the woman’s final flight leaving in __________
destination ? __________.
A. Chicago (W-Br) As long as it takes me to Los
B. New York Angeles, I’ll take it. My name’s Joan
Martin.
C. Los Angeles
(M-Am) Here’s __________
D. San Francisco __________, Ms. Martin. Please hurry
18.What does the man tell the to gate ten.
woman to do?
A. Present her identification
B. Call a travel agent
C. Check her luggage
D. Go to an airport gate

UNIENGLISHCENTER 64
19.Where did Flora stay in (M-Br) __________ __________,
Valencia? Flora! How was Spain?
(W-Am) Wonderful, thanks. I went with
A. In a hotel a few __________ we stayed at a
B. At a friend’s house __________ __________in Valencia for
two weeks.
C. With relative
(M-Br) Yeah. I really __________
D. At a campsite
Valencia when I visited my family there
20.When did the man go to __________ __________. Did you do a
Valencia? lot of sightseeing?
A. Last week (W-Am) Only a little, really. We were
B. Two weeks ago more interested in lying on the
__________ in front of the hotel every
C. Last year day. Once in a while, we did try out a
new __________ through.
D. Two years ago
21.How did Flora mainly spend
her vacation?
A. Trying new restaurants
B. Relaxing on a beach
C. Shopping in stores
D. Visiting her family
22.What is the woman looking for? (W-Br) __________ __________. I
think I left my glasses on a table when I
A. A book was here __________ __________. Has
B. A set of shelves anyone found them?

C. A new table (M-Br) Let me check our __________


__________ __________. No, I’m
D. A pair of glasses
sorry, they aren’t here
23.What does the man say he will
do ? (W-Br) Could you do me a favor and
call me if you find them? It’s difficult
A. Order the item for me to read without them and I have
B. Repair the item to __________ for an exam tonight.

(M-Br) I understand. After we reshelve

UNIENGLISHCENTER 65
C. Look for the item the books tonight, I’ll take a good
__________ __________ the library
D. Replace the item and call you if I find them.
24.Where does this conversation
probably take place?
A. In a library
B. In a furniture
C. In a classroom
D. In a doctor’s office
25.How do the speakers know each (M-Am) Clara, is that really you? I
other? didn’t __________ to see you here.
A. From the university (W-Br) Hi, Mark! I hadn’t planned to
B. Form a conference come, but I couldn’t miss our ten-year
__________. It’s really nice to be back
C. From the office on __________, isn’t it?
D. From a holiday party
(M-Am) It is. I can’t believe it’s been
26.How many years ago did the ten years since we’ve seen each other.
speakers last see each other ? __________ the time we won second
and third prizes in the __________
A. Two __________?
B. Five
(W-Br) Of course — that’s one of my
C. Ten best memories from our __________
days.
D. Fifteen
27.What does the woman say is
one of her favourite memories?
A. Learning to play chess
B. Attending a reunion
C. Winning a prize
D. Giving a presentation

UNIENGLISHCENTER 66
28.Where does this conversation (W-Br) Here are my __________. Can
probably take place? you give me an idea of when I should
__________ __________ to pick up the
A. In a theater car?
B. At an auto repair shop (M-Am) We need to rotate the tires,
change the oil, and perfom a safety
C. In a parking garage __________, so it’ll probably be ready
around 5 o’clock
D. At a car rental agency
29.When will the woman probably (W-Br) I can be back here by
return ? __________, but I’ll need to get to and
from my __________ today. Where can
A. At 2 o’clock I catch __________ __________?
B. At 4 o’clock
(M-Am) The bus stop is about two
C. At 5 o’clock hundreds meters down that way. There
are normally buses going into town
D. At 10 o’clock every 10 minutes or so. We’ll see you at
30.What does the man direct the the end of the day
woman to go?
A. To a bus stop
B. To an office building
C. To a store
D. To an apartment building

UNIENGLISHCENTER 67
ETS ACTUAL TEST 1

UNIENGLISHCENTER 68
UNIENGLISHCENTER 69
UNIENGLISHCENTER 70
NOTEPAD

UNIENGLISHCENTER 71
LESSON 7 – PART 3.2: CÂU HỎI VỀ SỰ VIỆC ĐƯỢC NHẮC ĐẾN
TRONG ĐOẠN HỘI THOẠI

I – LANGUAGE BUILDING
- Banquet - Fill out an application form
- Invitation - Place an ad (advertisement)
- Attendee - Be qualified for
- Retirement - Conduct an interview
- Venue - Work on
- Register - Specialize in
- Corporate event - Co-worker
- Company retreat - Colleague
- Tradeshow - Employee
- Foundation ceremony - Staff
- Adtend the training session - Predecessor
- Adjust the work shedule - Replacement
- Make a presentation/ Give a - Performance
presentation - Raise
- Call-off - Benefit
- Position - Cover
- Resume - Transfer
- Qualifications - Human resources
- Candidate - Personnel department
- Complete - Time sheet
- Submit - Take over the position
- Hire - Be in charge of
- Recruit - Contribute to
- Portfolio - Be promoted to
- Employment agency - Lay off
- Recommendation letter - Be dedicated to
- Job opening

UNIENGLISHCENTER 72
II – POWER LISTENING

1. What does the man want to do? M: I think it’s time I __________ my car
repair garage more aggressively. Do you
A. Buy a newspaper
think a __________ advertisement would
B. Have a car repaired be __________ ?
C. Advertise a business W: Maybe, but I really think you should
D. Start a new business set up a Website. I’m actually
__________ you don’t have one yet.
2. What does the woman suggest?
M: Hey, that’s a great idea. My friend
A. Buying a new car Ming __________ Internet sites. I think
I’ll give him a call.
B. Reading a newspaper
C. Surprising a new friend
D. Creating a Web site
3. Who does the man plan to
contact ?
A. A friend
B. A car repaired shop
C. A newspaper office
D. A government agency
4. Where does the woman want to W: Hello, I’d like to inquire if your
work? ___________ has any ___________ for
summer workers.
A. At a restaurant
M: Yes, we’re looking for ___________
B. At a beach
for the swimming pool, servers for the
C. At a health club ___________, and ___________
personnel for the grounds.
D. At a resort
W: I have ___________ as a lifeguard; can
5. What kind of employment
experience has the woman had? I come down this afternoon to fill out an
___________ ?
A. Fitness instructor
M: Sure, we’ll ___________ an interview
B. Lifeguard for you today, too; make sure you bring

UNIENGLISHCENTER 73
C. Gardener copies of your lifeguard ___________.

D. Restaurant server
6. What is the woman asked to
bring to her interview?
A. A résumé
B. An application
C. Proof of certification
D. Proof of insurance
7. What is the genaral topic of the W: Mark, I was surprised to hear that
conversation? you’re ___________ to move. Don’t you
like ___________ in the city ?
A. Housing
M: Actually, I do. The ___________ is
B. Employment
thar it’s very ___________ to rent an
C. Entertainment ___________ here. If I live outside the
city, it’ll be more ___________.
D. Transportation
W: Well, there are always a lot of
8. What does the man say about
living in the city? ___________ for apartments in the
newpaper. You should take a look at them.
A. There is too much traffic
M: That’s a good idea. My ___________
B. It is too expensive ___________ ends next month, so I nees
to find a new place to live soon.
C. It is easy to find a job
D. There are many apartments for
rent
9. What does the woman suggest
that the man do?
A. Look for a roomate
B. Find a new job
C. Check the newspaper
D. Sign a contract

UNIENGLISHCENTER 74
10.Who is the man shopping for ? M: Hi, ___________ ___________ does
your store ___________ tonight ? I need to
A. A coworker
get a ___________ ___________for my
B. A friend wife.

C. His child W: We’re open until seven P.M, and if you


can’t get to us today, we’ll be open at eight
D. His wife
o’clock ___________ ___________.
11.What time does the store close ?
M: I should be able to get there by seven –
A. At 5:00 but in case I don’t could you set aside an
item for me ? I can give you the
B. At 6:00
___________ ___________.
C. At 7:00
W: No, I’m very sorry. We can’t accept
D. At 8:00 reserve requeste over the phone because a
___________ ___________is required.
12.What does the customer ask the
salesperson to do?
A. Return a phone call
B. Reserve an item at the store
C. Keep the store open late
D. Provide a cash refund
13.What does the woman want? W: Excuse me, do you know where our
___________ is ? I’d like to order some
A. Some bread
___________.
B. A large table
M: No. I’m sorry. I’ll see if I can find him
C. Something to drink for you. Have you been waiting long?

D. A dessert W: ___________ ___________. But I can


see the restaurant is busy.
14.What does the man offer to do ?
A. Get some water
B. Go to a store
C. Bring a menu

UNIENGLISHCENTER 75
D. Find a waiter
15.How long has the woman been
waiting?
A. 2 minutes
B. 5 minutes
C. 10 minutes
D. 15 minutes
16.What type of business do the W: The ___________ ___________ just
speakers work for? called. Their ___________ is running late,
so they won’t be able to deliver the wood
A. A construction company
that we ___________ this afternoon.
B. A shipping company
M: So we aren’t getting the wood we need
C. A delivery service to ___________ the floors ?
D. A supply store W: Not today. But we could start putting
in the windows instead.
17.What problem are the workers
discussing ? M: Good idea. That way we won’t get too
far behind ___________.
A. Some wood is damages
B. Some roads are closed
C. Some workers will be late
D. Some supplies will not arrive
today
18.What will the workers probably
do next?
A. Hold a meeting
B. Work on a different project
C. Call the lumber company
D. Deliver a shipment

UNIENGLISHCENTER 76
19.Where does this conversation M: I can’t ___________ I’m here on time!
most likely take place? My car wouldn’t start this morning and I
had to take the ___________, which
A. At an office
usually takes about twenty minutes longer.
B. At a bakery
W: It’s a good thing you made it in. The
C. At a newsstand ___________ is in half an hour. I’m
wondering if we should use the
D. At a restaurant
___________ ___________ instead of this
20.How did the man get to work? office…

A. By car M: You’re right. Let’s take the


___________ and doughnuts and all the
B. By bus ___________ over there.
C. By train
D. On foot
21.When is the meeting scheduled
to begin?
A. In 5 minutes
B. In 10 minutes
C. In 20 minutes
D. In 30 minutes
22.Who is the woman? M: Jennifer, how’s your ___________
going? I heard your ___________ moved
A. A chef
to a new ___________ a couple of months
B. A server ago.

C. A restaurant owner W: Oh, Mike, hi. Yes, it’s near several


hotels and businesses in the city, so we get
D. A restaurant cashier
a lot of ___________ at lunchtime.
23.What does the woman say
M: Oh, that’s great. I should stop by
about the restaurant ?
sometime.
A. It is expensive
W: Please do! I just hired a new
B. It is busy at lunchtime ___________ from Italy last week, and
he’s been doing a great job so far.
C. It is located in a hotel

UNIENGLISHCENTER 77
D. It has a few open positions
24.What happened last week?
A. The woman hired a new
employee
B. The restaurant moved to another
location
C. The man went to Italy
D. The man started a new job
25.What are the speakers M: I hear that you saw Mountain Wild last
discussing? weekend. How did you like it ?

A. A movie W: It was ___________ – I went with


Martha from the ___________
B. A mountain hike
___________ ___________, and we both
C. A letter of recommendation thought it was the best movie we’d seen in
a while.
D. A job opening
M: So you would ___________ it? I’m
26.What does the man say he will
do on Friday? going on my day off this Friday.

A. Call the human resource office W: You should. As someone who love
hiking in the ___________, you would just
B. Set up a meeting with Martha love the breathtaking shots of Colorado..

C. Buy a book about Colorado


D. Go to see a film
27.What does the woman say
about the man?
A. He is the best candidate for a job
B. He enjoys the outdoors
C. He should go to Colorado
D. He should take an extra day off

UNIENGLISHCENTER 78
28.Where does this conversation W: I think I left my ___________
probably take place? ___________upstairs in my room. Do you
know when the first session starts?
A. At a coffee shop
M: It doesn’t start untill nine – thirty, so
B. At a train station
we still have time to go to the coffee shop
C. At a theater and get some ___________ with the rest of
the group.
D. At a hotel
W: That sounds like a good idea. Let me
29.What does the man suggest they
go up to my room and get my program,
do ? and I’ll meet you back here in the
A. Eat a meal ___________ in five minutes.

B. Buy a program
C. Make some copies
D. Review the schedule
30.When does the woman say she
will meet the man?
A. In 2 minutes
B. In 5 minutes
C. In 10 minutes
D. In 30 minutes

UNIENGLISHCENTER 79
ETS ACTUAL TEST 2

UNIENGLISHCENTER 80
UNIENGLISHCENTER 81
UNIENGLISHCENTER 82
LESSON 8 – PART 3.3: CÂU HỎI VỀ SỐ LIỆU CHI TIẾT TRONG ĐOẠN
HỘI THOẠI

I – LANGUAGE BUILDING

- Brand-new - Leave
- Inventory - Vacation
- Damage item - Sightseeing
- Refund - Destination
- Shipment - Accommodation
- Flyer - Single room
- Deliver - Itinerary
- Receipt - Land mark
- Defective - Xxx
- Release - Go on a trip
- Gift certificate - Make a reservation
- Out of stock - Travel agency
- Be sold out - Take a guided tour
- In stock - Departure
- Set up a display - Time table
- Arrange a display - Flight
- Get a full refund - Express train
- Exchange A for B - One way trip
- Made a last minute change to… - Round trip
- Charge an extra - Aisle seat
- Get cash back - Window seat
- Response - Room
- Inquiry - Tow

UNIENGLISHCENTER 83
- Reschedule - Heavy traffic
- Confirm - Fair
- Put A on hold - Give direction
- Connect A to B - Break down
- Get off the phone - Be schedule to do
- Make a telephone call - Be stuck in traffic

III – POWER LISTENING

1. What does the man ask Susan (M-Br) Susan, could you ___________ my
to do? nine o’clock client tomorrow? I need to
take my car to the repair shop.
A. Check a bus schedule
(W-Br) Sure, Mr.Miller. When is the
B. Change an appointment time earliest you could see him
C. Come in to work earlier
(M-Br) The ___________ told me that my
D. Call a car mechanic car should be ready by ten. So, let’s say
eleven. I’ll be in the office by then.
2. What will the man do tomorrow
morning? (W-Br) OK, I’ll ask him to come in at
eleven.
A. Have this car fixed
B. Visit a client’s office
C. Order some office supplies
D. Purchase a car
3. When will the man probably start
seeing clients tomorrow?
A. At 9 a.m
B. At 10 a.m
C. At 11 a.m
D. At 1 p.m

UNIENGLISHCENTER 84
4. What kind of company does the (M-Am) Thank you for ___________
man work for ? Riverside Gas COmpany. How may I
help you?
A. A telephone company
(W-Am) Hello, I have a ___________ about
B. A shipping company
my heating ___________ for this month. My
C. A heating gas company heating gas usage hasn’t changed recently,
but the charges seem much higher this
D. A bank time.

5. How does the man explain the (M-Am) It might be that the gas rates
change in the woman’s bill? went up in your ___________, but let me
look at your ___________ ___________to be
A. The price may have increased sure. What’s the account number?
B. The account number has changed
(W-Am) It’s two five one oh one. I live in
C. The company made a mistake Hutchins
D. The woman may have used more
heat

6. What information is the woman


asked to provide ?
A. Her address
B. Her account number
C. Her account balance
D. Her credit card number
7. What is being celebrated ? (W-Br) Joe, what time is everyone
meeting to ___________ Lucinda’s
A. The opening of a restaurant birthday?
B. A friend’s promotion
(M-Am) The ___________ is for six
C. The anniversary of a bank o’clock. Do you remember where the
___________ is?
D. A friend’s birthday
(W-Br) Yes, it’s on Elm Street, next to the
8. What time dose the celebration dry cleaner’s. But I need to go to the
begin? ___________ first, so I won’t get there
until six-thirty.
A. At 5:00
B. At 6:00

UNIENGLISHCENTER 85
C. At 6:30
D. At 7:00
9. Where will the woman go first?
A. To a conference room
B. To a dry cleaner’s
C. To a bank
D. To a restaurant
10.Why is Michael pleased ? (W-Am) Michael, I heard you won a
prize for your poetry. _________________!
A. He published an article
(M-Br) Thanks, I’m really honored that the
B. He won a prize Corwin Foundation regconized my
___________
C. He met his favorite poet
D. He taught his first writing class (W-Am) You must be very pleased. Are
you writing ___________ now?
11.What does Michael hope to do ?
(M-Br) Well, I’ve just started my first
A. Take a summer vacation ___________, and I hope to enroll in a
B. Sign up for a workshop writing ___________ this summer.

C. Apply for a scholarship


D. Join a reading club
12.What is Michael currently
working on?

A. A novel
B. A research paper
C. A news story
D. A poem
13.Where does this conversatuon (W-Br) I have a ___________ for you,
most likely take place? some bulding ___________- pipes and
tiles, I think. And I need someone to sign
A. At a post office for them. Is the site ___________
around?
B. In a music store

UNIENGLISHCENTER 86
C. In a plumbing supply shop (M-Am) He’s not here today. I’m the
___________ ___________; I can sign the
D. At a construction site form for you.
14.What does the man offer to do ? (W-Br) Great, thanks. Where would you
A. Sign a form like me to ___________ these boxes?

B. Unload a truck (M-Am) The pipes go over to that red


building. And put the tiles ___________
C. Make a delivery ___________.
D. Look for the site manager
15.What will be delivered over by the
red building?
A. Tiles
B. Stamps
C. Pipes
D. Doors
16.Who most likely is the man? (W-Am) Excuse me. ___________
___________does it cost to park here?
A. A parking attendant
(M-Am) It’s five dollars for the first
B. A bank manager
hour, and then three dollars for every
C. A museum diretor additional hour.

D. A bookstore clerk (W-Am) That’s kind of ___________. I just


need to run into the ___________ —
17.Where is the woman going ? they’re holding some ___________ I
ordered. It’ll only take a few minutes.
A. To get some money
B. To walk in the park (M-Am) Well, the parking area behind
the ___________ ___________ only
C. To pick up an order charges two dollars for a half hour. You
should be able to find a space there.
D. To visit a museum
18.Why does the man recommend a
different location?
A. It is in a more convenient place
B. It offers lower prices

UNIENGLISHCENTER 87
C. It has more free space
D. It can fill special orders for
customers
19. What color car is in stock now? (M-Br) Is this car ___________ in silver or
red?
A. Blue
B. Red (M-Am) We only have this blue one
______________ right now, but we can
C. Silver order a silver or red one, too. You can
also have sport stripes added for an
D. Black extra 500 dollars.
20. How much does it cost to add
stripes to the car? (M-Br) I think I’d like to order a
___________ one, then, but no stripes. I
A. $100 heard that silver-colored cars hold their
value very well.
B. $200
C. $400 (M-Am) Great, then let’s step into my
___________ and take care of the
D. $500 ___________. We should have your
___________ for you in about
21. How soon will the customer ___________ ___________
receive his car?
A. In one week
B. In two weeks
C. In four weeks
D. In five weeks
22. What is the man’s problem? (W-Am) Robert, have you received your
new ___________ ___________? I got mine
A. He cannot find the security office on Friday.
B. He cannot find the exit
(M-Am) Friday? I still don’t have mine. I
C. He cannot open the door have to ask a ___________ guard to open
the door for me everyday.
D. He has too much to do
23. When did the woman receive her (W-Am) Maybe you should speak to
access card? someone in the security office about
issuing you a ___________.
A. On Tuesday
(M-Am) You’re right. I’ll go there

UNIENGLISHCENTER 88
B. On Wednesday ___________ ___________.
C. On Thursday
D. On Friday
24. What does the woman suggest the
man do?
A. Wait for the security guard
B. Use the back door
C. Go to lunck earlier
D. Contact the security office
25. Where does this conversation (M-Br) How would you like ___________
most likely take place? ___________today? Would you like just a
trim, or would you like to try a
A. At a factory ___________ ___________?
B. At a hair salon
(W-Br) I’d like to try a new shorter style,
C. At a newsstand like this ___________ in this ___________.
Do you think you can make my hair look
D. At a clothing store like this?
26. What does the woman want to do?
(M-Br) No problem. I’ve just ___________
A. Travel overseas a ___________ ___________ in Paris on
short hairstyles, and that was one of the
B. Remodel a building ones we learned.
C. Buy some clothing
D. Change her hairstyle
27. Why did the man go to Paris?
A. To write a magazine article
B. To go sightseeing
C. To meet a client
D. To take a training course

28. What is the man’s problem? (M-Am) Hey, Vicki. Have you had any
___________ using your computer this
A. He lost his room keys morning? I can’t log on to mine.

UNIENGLISHCENTER 89
B. He was late coming to work (W-Am) Our ___________ expired last
night. You have to go to the technical
C. He cannot access his computer ___________ ___________to have a new one
D. He forgot his computer password set up.
(M-Am) OK… that’s down on the first
29. What will the man probably do floor by the ___________ ___________, right?
next ? (W-Am) It’s on the first floor, but you need
to turn left at the ___________
A. Go to the machine room ___________and go all the way down that
B. Get a new password hailway. It’s right next to the
___________ ___________.
C. Call the reception desk
D. Ask a security guard for help
30. Where is the technical services
office located?
A. On the second floor
B. Next to the reception desk
C. Near the security office
D. By the stairs

UNIENGLISHCENTER 90
ETS ACTUAL TEST 3

UNIENGLISHCENTER 91
UNIENGLISHCENTER 92
UNIENGLISHCENTER 93
ETS ACTUAL TEST 4

UNIENGLISHCENTER 94
UNIENGLISHCENTER 95
UNIENGLISHCENTER 96
LESSON 9 – PART 4 – MESSAGE

I – EXAMINATION

71. Why will the speaker be late for Hi, Robin, it’s John. I’m calling to let you
work? know that I’ll be a little late getting to the
office today. My water heater suddenly
A. His car has broken down
stopped working, so I called a repairperson
B. He is waiting for a repairperson to come and fix it. I’m waiting for him to
get here. Unfortunately this means I won’t
C. He has a medical appointment
be able to attend the sales meeting at nine
D. His train has been delayed thirty. There are copies of the meeting
agenda on my desk – could you hand them
72. What is the speaker scheduled to do at out at the meeting? Thanks so much.
9:30?

A. Open a store

B. Train new staff

C. Make a delivery

D. Attend a meeting

73. What does the speaker ask the listener


to do?

A. Distribute some documents

B. Type a report

C. Provide driving directions

D. Buy some supplies

UNIENGLISHCENTER 97
II – LANGUAGE BUILDING

- Bulletin board - Engine trouble

- Board of director - Proceed to

- At short notice - Shut down

- Security policy - Patron

- ID badge - Register

- Check in - Sale representative

- Work assignment - Grocery

- Modify - Retail outlet

- Maintenance - Shopping complex

- Installation - Check out

- Expand - Customer service counter

- Service person - Forman

- Replace - Performance

- Energy efficient - Exhibition

- Safety measures - Ticket counter

- Attention - Complimentary

- Flight - Audience

- Destination - Demonstration

- Arrival board - Sound equipment

- Due to - Auditorium

UNIENGLISHCENTER 98
III – MINI TEST

A. Recorded message ( Tin nhắn được ghi âm)

71. Why is the dentist’s office closed Hello, you’ve ___________ Dr.
today? Samson’s dental clinic. Our office is
(A) It is being relocated currently closed for ___________ and
(B) It is undergoing renovations won’t be open until next Monday. If you
(C) It is being inspected are calling to make an ___________,
(D) It is observing a national holiday please leave your name and
___________ ___________ after the
72. What is said about the clinic?
beep. Also, I’d like to remind you that
(A) It will reopen this week
our office hours will be changed as of
(B) It will be closed on Sundays
next week.
(C) It will be open until 9pm during this
Please note that when ___________ we
week
will be open from 9 am to 7 pm every
(D) It will hire more dentists in the near
day ___________ for Sunday. Should
future
you need immediate assistance, please
73. What are the listeners advised to call 4451100. Thank you.
do in case of emergency?
(A) Visit the clinic
(B) Stay on the line
(C) Dial another number
(D) Contact another clinic

74. What is stated about the library? Hello. You’ve ___________ Underwood
(A) It will be closed tomorrow Community Library. We’re currently
(B) It is open 7 days a week ___________. We’re open from 9 to 8
(C) It has an online booking system every day except for Monday. For
(D) It will be celebrate its 10th ___________ ___________, please call us
anniversary soon back during our regular hours. If you
wish to ___________ a book, you can
75. What will happen on April 10th?
visit our online reservation service at
(A) The online reservation system will
www.underwoodbooks.com. Through
be installed
the website, you can also refer to the
(B) The library will be closed for an
listings of ___________ ___________.
inspection

UNIENGLISHCENTER 99
(C) The new facility will open to the Finally, please note that children’s
public reading room that can ___________ as
(D) The children’s reading room will be many as 60 children will soon be open.
refurbished The ___________ ___________
___________is scheduled to
76. What are listeners encouraged to th
___________ ___________on April 10
do?
and everyone is welcome to attend.
(A) Show up at the event
(B) Return books on time
(C) Pay any late fees by April 10th
(D) Leave a message

77. What is the annoucement mainly Hello, this automated telephone


about? ___________ is to remind all TM
(A) The obsolete workstations electronics ___________ that our
(B) The new payroll system computer database system will be
(C) The training instructors ___________ over the weekend. As a
(D) The upcoming upgrades result, we will not be allowed to access
our ___________ ___________, including
78. When will the scheduled work be
our corporate email system, starting
finished?
from 6 pm on Friday, with the entire job
(A) On Friday
___________ to be ___________ on
(B) On Saturday
Sunday. Please don’t ___________ to
(C) On Sunday
back up all important data on your
(D) On Monday
computer before you ___________the
79. What will employees receive office on Friday. Next Monday, Jane
before 2 pm on Monday? Parker from the tech ___________ will
(A) A new code give you more detailed instructions n
(B) A laptop computer how to dea with the new system at 2pm.
(C) A payment Also, you will receive your new
(D) An ID badge ___________ and password before the
afternoon training.

80. Why will the library be closed Hello. You’ve reached the Westwood
from July 10? public ___________. I’m sorry that
(A) To add a wing we’re currently closed. Our regurlar
(B) To improve the facilities hours of ___________ are from 9 am to 8
(C) To move to Jackson County pm every day except Monday. Should

UNIENGLISHCENTER 100
(D) To replace desks and chairs you need any immediate ___________,
please call our after-hours line at 690-
81. What are listeners asked to do in
009. If you are a ___________ of our
case of emergency?
library, please note that we will be
(A) Stay on the line
closed from July 10 to August 5 for
(B) Leave a message
extensive ___________, including
(C) Visit the front desk
knocking down the dividing walls and
(D) Call another number
replacing the flooring. However, you
82. How can listeners use other public can still borrow books and ___________
libraries? ____ various programs at other public
(A) By laying extra fees libraries in Jackson County. All you
(B) By getting a temporary pass need to do is present your library card at
(C) By calling an after-hours line the counter. Once again,we’re sorry for
(D) By showing a library card the ___________.

B. Voice mail message ( Tin nhắn thoại)

71. What is the purpose of the call? Hello, Mr. Duke. This is Sean Lee from
(A) To ask questions about the the Manchester Book Club. I’m calling
upcoming conference to ask whether you can ____________
(B) To invite the writer to an event ____ the Manchester Reading Festival
(C) To purchase the lastest best seller next month. We’d like you to read some
(D) To interview a celebrity __________ from your lastest bestseller,
The Revenge, and share _________
72. How long will the event last?
with our audience on the main stage. I
(A) 1 day
think it would be a _________
(B) 3 days
_________for the public to explore the
(C) 7 days
world of your writing. The event
(D) 9 days
_________ ____ _________ in the
73. What does the speaker want Mr. Global Cultural Center from May 3 to
Duke to do? May 9 and I hope you’ll be able to join
(A) Reserve a seat in advance us on the first day of the _________. To
(B) Give directions to the venue find out more about the event, please
(C) Join the event on May 3 visit www.manchesterreading.com or
(D) Talk about his future plans call me at 450- 9900 anytime you want.
I’m _________ _________ ___hearing

UNIENGLISHCENTER 101
from you soon.

74. What is the purpose of the call? Hi, Stewart! This is Helen Haywood
(A) To convince Stewart to deliver a from the _____________ _________.
talk I’m calling to let you know that we are
(B) To promote an upcoming event going to start a _________ ____
(C) To create new marketing techniques _________ call Money Talks from next
(D) To correct a mistake Monday. We will _________ renowned
experts on stocks, bonds, and real estate
75. In which area is Mr. Cook an
to the stage and give you the chance to
expert?
hear about their lastest _________
(A) Finance
_________. Also, don’t be surprised!
(B) Law
We finally convinced the famous
(C) Technology
_________ _________, Charles Cook,
(D) Time management
to deliver a talk about how to invest in
76. What does the speaker want? stocks on Monday at 7 pm. A more
(A) An early response _________ _________ is available on
(B) A fax message our website at www.financialsecret.com.
(C) A deposit Early registration is _________, as
(D) A membership fee attendance is limited up to 200
participants per lecture. If you’re
_________ ___ _________, please give
me a call as soon as possible. Thank
you.

C. Telephone message ( Tin nhắn điện thoại)

71.What is the purpose of the Hello, Mr Santos. This is Steven Chan. I


message? work with Crawford Relocation
(A) To inquire about a job applicant Services. I’m _________ _________
(B) To ask about accommodations an apartment for a _________ who is
(C) To offer help with relocation moving to the _________ in two
(D) To inform a client of an available weeks. Last week you ran an ad in the
apartment paper aboutt an apartment for rent on
Manor Drive. Is it still _________?
72. How does the speaker ask to be
Could you please call me at 217-555-
contacted?

UNIENGLISHCENTER 102
(A) By telephone 3222 with _________ about that
(B) By e-mail apartment or any available rental space
(C) By letter in that ____________? Thank you, and I
(D) By fax hope to hear from you soon.

73. What will Mr. Santos probably


do?
(A) Attend an event on Manor Drive
(B) Invite Mr. Chan to his house
(C) Return Mr. Chan’s call
(D) Submit an application form

74. Where does the caller probably Ms. Farino, this is Diane from the
work? Pendale Medical Center. I’m calling to
(A) At a travel agency let you know that your _____________
(B) At an airport with Dr. Morris this afternoon will have
(C) At a doctor’s office to be _____________. Dr. Morris was at
(D) At a hotel a _____________ in Dallas, and his
flight was delayed – he won’t be back
75. What is the cause of the problem?
until this evening. Could you please
(A) A tour was canceled
give me a call here at 555-0192 so we
(B) A room is not available
can _____________ your appointment?
(C) A conference date has changed
I’m very sorry for the late notice, Ms.
(D) A flight was delayed
Farino, and we _____________ to give
76. What is Ms. Farino asked to do? you the earliest available appointment.
(A) Make a new appointment
(B) Confirm a conference schedule
(C) Provide contact information
(D) Use a different method of
transportation

77. Where most likely does the (W-Am) Hi, Mr. Reed, this is Margo
speaker work? from Tweed Menswear. I’m calling to
(A) At a stationery store let you know that we’re _____________
(B) At a toy store with the alterations on the
(C) At a print shop _____________ you purchased. The
(D) At a clothing shop shop will be closed next week, so if

UNIENGLISHCENTER 103
78. Who is the speaker calling? you’re unable to pick them up by
(A) An employee Friday, you’ll have to wait until we
(B) A supplier _____________ on September first.
(C) A customer Thank you again for your
(D) A delivery service _____________, and we hope to see
you soon.
79. According to the speaker, what
will happen next week?
(A) An item will be ordered
(B) A sale will be held
(C) A bussiness will be closed
(D) A catalog will be mailed

80. Where does the speaker work? (M-Au) Hi, this _____________ is for
(A) At a dry cleaner Jim Brenner. This is Anthony from
(B) At a bank Anthony’s Dry Cleaners. I’m
(C) At a hotel _____________ to let you know I found
(D) At a clothing shop a _____________ _____________ in
the pocket of the brown suit jacket you
81. What was found?
brought in yesterday. Your
(A) A set of keys
_____________won’t be ready until
(B) A digital camera
next week, but I thought you might want
(C) A mobile phone
to get your card back sooner than that.
(D) A credit card
When you _____________ ___, please
82. What will the listener have to have your suit claim check with you so
show to pick up the item? that we know that we’re
(A) A hotel receipt _____________ _____
(B) A claim check _____________to the right person.
(C) A bussiness card Thanks.
(D) A passport

UNIENGLISHCENTER 104
ETS ACTUAL TEST 1

UNIENGLISHCENTER 105
UNIENGLISHCENTER 106
UNIENGLISHCENTER 107
ETS ACTUAL TEST 2

UNIENGLISHCENTER 108
UNIENGLISHCENTER 109
UNIENGLISHCENTER 110
LESSON 10 – PART 4 – ANNOUCEMENT & INTRODUCTION

I – EXAMINATION

77. What will happen next week? Attention, employees. This is a reminder
that repaving work will be taking place all
A. A fee will be increased
next week in the west parking area.
B. Menus will be posted Because of the reduced number of available
parking spaces, employees are encouraged
C. A parking area will be repaved
to share rides for the duration of the
D. Hours of operation will be extended repaving project. A sign-up sheet for
carpooling has been posted in the cafeteria.
78. What are employees encouraged to It you’d like to arrange to share a ride, stop
do? by at lunch and add your name and phone
A. Share transportation to work number to the list.

B. Work additional shifts

C. Contribute to a newsletter

D. Join a planning committee

79. Where can the sign-up sheet be


found?

A. In the lobby

B. In the cafeteria

C. In the conference room

D. In the mail room

UNIENGLISHCENTER 111
II – LANGUAGE BUILDING

- Contact - Appointment
- Extension - Reschedule
- Response to - Delay
- Inquiry about - Shop hour
- Receipt - Office hour
- Policy - Business hour
- Manufacture - Hour of operation
- Discount - Confirm a reservation
- Bill - Make a reservation
- Statement - Reach
- Purchase - Beef
- Shipment - Hotline
- Merchandise - Tone
- Retailer - Connect
- Place an order - Star key
- Customer service representative - Pound key
- Recall - Voice mail
- Replace - Automated message
- Fix - Leave a message
- Manufacturing floor - Take a message
- Inconvenience - Return one call
- Purchase price - Get back to
- Mailing address

UNIENGLISHCENTER 112
III – POWER LISTENING

A. ANNOUNCEMENT

77. What is the purpose of the _____________ shoppers. It’s 9:45 pm,
announcement? and our _____________ _________
(A) To inform customers that the store will be closing soon. Our store will
will be closed soon _____________ tomorrow morning at
(B) To invite customers to buy a variety 10. If you have any products you would
of ballons like to buy, please proceed to one of our
(C) To encourage customers to use a gift _____________ _____________ now.
certificate instead of cash If you haven’t found any Christmas
(D) To urge customers to buy Christmas presents for your family members yet,
gifts for their co-workers why don’t you give them our ________
_____________? We have both 10 and
78. When will the store close
20 dollar gift certificate available at our
tomorrow?
_____________ _____________
(A) 9:45
_____________ on the second floor.
(B) 10:00
And I’d like to remind you that our
(C) 11:00
bussiness hours will be extend to 11 pm
(D) 12:00
from tomorrow. Finally, _________
79. What is said about gift _____________ to pick up your free
certificates? balloon for your kids at the main gate
(A) They cost 5 dollars when you walk out of our store. Thanks
(B) They will be distributed at the main for __________ at Dave’s today.
gate
(C) They are available at the customer
service desk
(D) They will go on sale tomorrow.

80. Who is the announcement for? Attention, all factory workers. This is a
(A) New emplyees _____________ that our ____________
(B) Plant workers department will be conducting a routine
(C) Inspectors _____________ on all production
(D) Electricians equipment from 9 am to noon today.

UNIENGLISHCENTER 113
81. According to the speaker, what Although our normal business
will take place this morning? _____________ are not affected,
(A) New safety regulations will be electrical services will be shut down for
annouced safety issus. As a result, all
(B) All elevators will be inspected _____________ except the ones in the
(C) Electrical services will be improved main factory building won’t operate
(D) A regular check will be done during the inspection. If you have any
_____________ or _____________,
82. Why most likely won’t elevators
please call our maintenance office at
be in operation?
extension 331. Thank you for your
(A) They are too old
cooperation.
(B) They will be replaced
(C) The factory will not have electricity
(D) They will be fixed

83. When is the new flight scheduled Ladies and gentle men, we regret to
to arrive at its destination? _____________ yout that the 10:00 AM
(A) 10:00 AM National Airlines flight from
(B) 1:00 PM Philadelphia to Seattle has been
(C) 2:00 PM _____________. There is another flight
(D) 7:00 PM scheduled to leave from gate 3A here in
Philadelphia at 1 PM, and
84. What is National Airlines
_____________ in Seattle at 7:00 this
offering?
evening, with a short stopper in
(A) A refund for tickets
Chicago. We _____________ _____ the
(B) A free trip to Seattle
inconvenience. Each _____________ on
(C) A discount on a future flight
the canceled flight will receive a
(D) An earlier flight to Philadelphia
_____________ for 50 dollars off a
85. Where will the new flight depart future flight anywhere in North
from? America. Again, the _____________
(A) Gate 3A from Philadelphia to Seattle via Chicago
(B) Gate 7A will leave gatge 3A at 1:00 PM and
(C) Gate 10A arrive in Seattle at 7:00.
(D) Gate 11A

UNIENGLISHCENTER 114
86. Who is the conference intended (M-Au) I’m pleased to _____________
for? everyone to the Journalism Today
(A) Advertising professionals conference. We’re happy to have
(B) Software developers _____________ joining us from a
(C) Physicians variety of _____________
(D) Journalists _____________. But before I
_____________ the keynote speaker, I
87. What does the speaker say has
have a note about a change to one of
changed?
today’s programs. The Medical
(A) The title of a presentation
Journalism panel has been
(B) The starting time of a session
_____________ to start later in the day.
(C) The location of dining facilities
It will be held at 3 PM and will still be
(D) The keynote speaker
in the Parkview Roon. More
88. According to the speaker, where is _____________ about today’s programs
more information available? is available at the _____________
(A) On a Web site _____________.
(B) In a brochure
(C) On a notice board

B. INTRODUCTION

71. Where is the speaker? I’d like to welcome you to the Blue
(A) At a hotel Water Hotel! As _____________, we
(B) At a restaurant offer each guest a _____________, full
(C) At an electronics store breakfast for every day that you spend
(D) At a sports center with us; lunch and dinner are not
included. Each room is _____________
72. What is stated in the talk?
for Internet access. You just need to
(A) Room service is available
_____________ a computer access card
(B) Evening entertainment is provided
from the front desk. And remember that
(C) The fitness center is closed
our special group rate includes free use
(D) Free computer software is offered
of the hotel’s _____________
73. What is included in the group _____________, swimming pool, and
rate? free tickets to one of our nightly popular
(A) All meals _____________ _____________.
(B) Internet access

UNIENGLISHCENTER 115
(C) A concert ticket
(D) Swimming lessons

77. What is the topic of the (W-Br) I’d like to thank everyone for
workshop? attending this _____________ today.
(A) Increasing customer satisfaction My name’s Valerie Fairchild, and in this
(B) Setting effective goals _____________ we’ll be talking about
(C) Becoming a better supervisor how to become an affective public
(D) Improving public speaking skills speaker. In just a few minutes I’ll be
_____________ you through a series of
78. What concern does the speaker
activities designed to build your
express?
_____________ in front of large groups.
(A) There will not be enough seats for
But, I see that we seem to be short on
the participants
chairs. Please find a seat quickly so we
(B) The location of the session has
see how many more chairs we need –
changed
then I’ll _____________ the head of
(C) The session will last longer than
_____________ about having more
scheduled
brought into the room.
(D) Participants have not received the
class materials

79. Who des the speaker say she will


contact?
(A) The conference organizer
(B) The maintenance supervisor
(C) An advertising executive
(D) A human resources manager

UNIENGLISHCENTER 116
ACTUAL TEST 3

UNIENGLISHCENTER 117
UNIENGLISHCENTER 118
UNIENGLISHCENTER 119
ACTUAL TEST 4

UNIENGLISHCENTER 120
UNIENGLISHCENTER 121
UNIENGLISHCENTER 122
LESSON 11 – PART 3 & 4: HAI LOẠI CÂU HỎI MỚI

I – EXAMINATION

41. Where do the speakers most likely


work? M. Hi Tricia, I was just looking over the
advertisement that you finished for Jenning
A. At an advertising firm
Bank, and you did a wonderful job.
B. At a bank
W. Thanks, it was a great campaign to work on.
C. At a law office
M. Would you consider joining my team for
D. At a travel agency our new contract with Circle Bookings?
They’re a popular travel Web site, and they’re
42. What does the woman mean when she looking to expand their market with some
says, ―I really can’t say‖? advertisements.
A. She is not allowed to repeat certain
W. It sounds exciting, but at this point I really
information
can’t say. I have a meeting with my manager
B. She cannot make a commitment yet. tomorrow morning though.

C. She should leave for an appointment M. Ok. Would it help if I stop by your
meeting and go over the details with both of
D. She has to correct some errors in a report you? This way, your manager will know what
43. What does the man propose? the project entails.

A. Making travel arrangements

B. Preparing a contract

C. Joining a meeting

D. Reviewing a report

UNIENGLISHCENTER 123
Admission Price per Person W. Tom, there’s a new theater production
opening at the Town Playhouse, and some of
University student $8
us from work are planning to go. Are you
Group of 10 or more $12 interested?

Member $15 M. Sure, I’ve heard it’s a good play. How much
do tickets cost?
Nonmember $20
W. It depends. Look, here’s the information. We
already have some more than ten people
62. What type of event are the speakers interested, so we should qualify for that price
discussing? M. That’s certainly reasonable. Would that be
A. A theater performance for this weekend?

B. A museum exhibit opening W. Yes, after work on Friday. Do you want to


go?
C. A photography workshop
M. Sure, are you going to order the tickets?
D. A live music concert
W. No, Mary Jones in the finance department is.
63. Look at the graphic. What ticket price You could give her a call and let her know to
will the speakers probably pay? include you.
A. $8 B. $12 C. $15 D. $20

64. What does the woman suggest the man


do?

A. Leave work early

B. Call a coworker

C. Pay with credit card

D. Rent some equipment

UNIENGLISHCENTER 124
II – POWER LISTENING

TASK 1

77. Where most likely is the speaker? W. Hi, it’s your neighbor, Indira. My
return fight was scheduled to leave in an
A. At her house
hour and they just announced it’s been
B. At an airport cancaled. I’m standing in line right now,
waiting to talk to a ticket agent. I’m so
C. In a taxi tired of traveling, but it looks like I won’t
D. On a train make it home this evening. Can you
believe it? Anyway, I have a favor to ask.
78. What does the speaker mean when she says, Cou you go by my house after work? A
“Can you believe it”? package was supposed to arrive today,
A. She is annoyed and I’d rather it didn’t just sit in front of
my door overnight. I really appreciate it!
B. She is excited

C. She is embarrassed

D. She is confused

79. What does the speaker ask the listener to do?

A. Look a door

B. Check an address

C. Meet a colleague

D. Pick up a package

UNIENGLISHCENTER 125
47. Where most likely is the woman? W. Welcome to Lehmann Law Offices.
How can I help you?
A. A telephone operator
M. I’m with Hannover Express Shipping,
B. A post office clerk
And I have a package her for Mr. Dennis
C. An office receptionist Lehmann. Is he available to sign this
form?
D. A sales associate
W. I can sign this for him, if that’s all
48. Why is the man visiting the office? right
A. To attend a training session
M. Sorry, This package must be signed
B. To repair some computers for by Mr. Lehmann himself

C. To apply for a job W. Well, Mr. Lehmann’s in a meeting


with clients right now.
D. To make a delivery
M. Ok, I’ll stop by later.
49. What does the speaker mean when she says,
“Mr. Lehmann’s in a meeting with clients right
now”?

A. Mr. Lehmann has a document she needs

B. A meeting room cannot be used

C. Mr. Lehmann is not available

D. An interview had to be cancelled

UNIENGLISHCENTER 126
FROM SUBJECT

Mike Collins Budget Report M. Hey Jenny, is the Internet working on


your computer?
Jared Huber Sales Projection Assistance
W. Yeah, I’m not having any issues with
Darla Conference Agenda it.
Janice West Canceled: Seminar M. Well, I can’t connect to it, so I can’t
see my email. Did the one with the latest
budget report come yet?
68. Why is the man unable to access his
W.Hmm … Let’s see. Here it is. Do you
e-mail?
want me to send a response?
A. His password has expired
M. That won’t be necessary, but could
B. His Internet connection is not working. you print it out for me? I need a copy of
the budget report for the meeting this
C. He forgot to update some solfware
afternoon.
D. He left a power cord at home

69. Look at the graphic. Who sent the email the


speakers are referring to?

A. Mike Collins

B. Jared Huber

C. Darla

D. Janice West

70. What does the man ask the woman to do?

A. Call for technical assistance

B. Prepare for training materials

C. Print out a document

D. Review some sales figures

UNIENGLISHCENTER 127
Length of Contract Cost per month M. Hi, I’m looking for an Internet
provider, so I thought I’d stop by to find
3 months $40.00 out about your service plans.
6 months $30.00 W. Certainly. We offer the best prices in
1 year $20.00 the area. As you can see from this chart,
the longer your chart, the longer your
2 years $10.00 contract is, the lower the monthly cost

M. But what if I have to cancel the


contract before it ends?
68. According to the woman, when is an extra fee
charged ? W. Well. We do charge an extra fee for
that
A. When new software is installed
M. Umm… I’m being transferred
B. When a contract is canceled early
overseas in about a year. So I don’t want
C. When a customer transfer to a new location the two year – plan. But I do want the
lowest possible price.
D. When a payment is overdue
W. Ok then the one year plan would be
69. What does the man say he will do next year?
best . Would you like to sign the contract
A. Move overseas now ?

B. Complete a training program M. Sure, let’s do that

C. Purchase another computer

D. Sign a longer contract

70. Look at the graphic. How much has the man


agreed to pay per month ?

A. $40.00

B. $30.00

C. $20.00

D. $10.00

UNIENGLISHCENTER 128
MINI TEST

UNIENGLISHCENTER 129
UNIENGLISHCENTER 130
UNIENGLISHCENTER 131
UNIENGLISHCENTER 132
GRAMMAR
PART 5 & 6:
LESSON 1A – NOUNS
A. Các loại danh từ:
1. Danh từ đếm được:
a. Danh từ số ít: thường là những danh từ đếm được và luôn có từ hạn định đứng trước ( mạo từ,
tính từ sở hữu, chỉ định từ, từ chỉ số lượng)
- Mỗi danh từ chỉ có 1 từ hạn định đi kèm. Nhưng một danh từ có thể có nhiều tính từ cùng bổ
nghĩa cho nó.
Từ hạn định: I bought a chair
Tính từ: I bought a small round wooden chair.
- Khi danh từ có cả từ hạn định lẫn tính từ đi kèm thì từ hạn định sẽ đứng trước tính từ
Ex: This new product is available next month.
b. Danh từ số nhiều: là những danh từ được hình thành từ danh từ số ít và thường được nhận biết
bằng đuôi S, ES, IES.
Ex: a reservation – reservations ; a fax – faxes ; a city – cities
- Một số danh từ số nhiều đặc biệt:
Man – men
Woman – women
Child – children
Tooth – teeth
Foot – feet
* Những danh từ đếm được hay xuất hiện trong bài thi Toeic:
Purpose Day Challenge Survey
Attempt Meeting Agreement Problem
Direction Argument Instruction Facility
2. Danh từ không đếm được: Đối với danh từ không đếm được, ta có thể dung mạo từ THE làm từ
hạn định đứng trước, lưu ý không được dung a/an
* Những danh từ đếm được hay xuất hiện trong bài thi Toeic:
Baggage - Luggage Advertising Clothing Progress
Merchandise Traffic Satisfaction Economics
Machinery Equipment Furniture Attendance
Advice Information Knowledge News
Experience Evidence

UNIENGLISHCENTER | LESSON 6: GIỚI TỪ 133


B. Các loại từ hạn định và cách dùng:
Danh từ đếm được Danh từ không đếm
Từ hạn định
Số ít Số nhiều được
His His employee His employees His information
This / that This employee This information
These/those These employees
Many/ a few / few Many employees
Much / a little / little Much information
Each / every Each employee
Some, most, all Some employees Some information

C. Hậu tố của Danh từ


Dưới đây là 12 hậu tố của DANH TỪ thường gặp trong bài thi TOEIC.
Suffix Example Meaning Suffix Example Meaning

AssistANCE Sự trợ giúp DepartURE Khởi hành


ANCE
URE
/ENCE
ConfidENCE Sự tự tin ExposURE Phơi bày
Trợ lý
NESS CompetitiveNESS Tính cạnh tranh ANT/ER AssistANT

CY/ACY AccurACY Sự chính xác SHIP MemberSHIP Tư cách hội viên

MENT DevelopMENT Sự phát triển ITY ResponsibilITY Trách nhiệm

SION
DeciSION Sự quyết định HOOD ChildHOOD Thời thơ ấu
/TION

Danh từ chỉ người hoặc nghề nghiệp thường kết thúc bằng các hậu tố: -er, -or, -ee, -
ian, -ist, - ent, - ant, -ast
-ant -Or, -er -ee
accountant supervisor employee
Participant contributor trainee
applicant contractor interviewee
assistant interviewer Refugee

UNIENGLISHCENTER | LESSON 6: GIỚI TỪ 134


*CHÚ Ý: Một số danh từ có kết thúc bằng đuôi của tính từ như:
Đuôi TIVE
- Alternative (n): người, vật thay thế
- Incentive (n): sự khuyến khích, thúc đẩy
- Initiative (n): Sáng kiến. Ex: drug treatment initiative- sáng kiến xử lý thuốc phiện
- Representative (n) người đại diện, đại lý. Ex: We have two representatives in this
city- chúng tôi có 2 đại lý ở thành phố này.
- Perspective(n): quan điểm, cách nhìn, cái nhìn. Ex: My father gave me a whole new
perspective on life- Bố của tôi đã cho tôi một cái nhìn hoàn toàn mới về cuộc sống.
- Objective (n) : Mục tiêu mục đích
Đuôi AL
- Potential: tiềm năng
- Professional : chuyên gia. Ex: he is a professional - anh ấy là một chuyên gia
- Individual: cánhân. Ex: I need an individual to go with me right away - tôi cần
một cá nhân đi cùng tôi ngay lập tức
- Approval : sự phê chuẩn, sự đồng ý. Ex: Once I get the approval from the boss, I‘ll
distribute the report to everyone in the accounting department- Ngay khi tôi có được sự phê
chuẩn của ông chủ, tôi sẽ phân phát bản báo cáo cho tất cả mọi người trong bộ phận kế toán.
- Proposal : sự đề nghị, lời đề nghị. Ex: Mr.Han asked you to turn in your proposal-
Ông Han đã yêu cầu anh nộp bản đề xuất.
- Arrival : sự đến, sự tới nơi. Ex Call me shortly after your arrival at the airport- gọi
cho tôi ngay sau khi bạn đến sân bay
D. Vị trí của Danh từ
1. Danh từ đứng đầu câu, có chức năng làm chủ ngữ của câu.
INSTRUCTIONS are given to every single employee to make sure they get things right.
PRODUCTS returned to the store after purchase must be checked for flaws.
2. Danh từ làm tân ngữ đứng sau động từ hay sau giới từ
The management decided to discontinue PRODUCTION until further notice. The
library will be accepting DONATIONS of used textbooks.Because the menu was
not in his native language, the visitor asked the waiter for GUIDANCE.
3. Danh từ đứng sau từ hạn định
Most INVITEES are from European countries. The contract contains a PROVISION.

UNIENGLISHCENTER | LESSON 6: GIỚI TỪ 135


The APPLICANTS had so many qualifications.
This ORDER comes from one of our loyal customers.
4. Danh từ đứng sau hoặc trước một danh từ khác để tạo thành một danh từ ghép. Danh
từ đứng sau tính từ, sau ENOUGH
The research and DEVELOPMENT department just came up with a new fresh idea.
Successful APPLICANTS will be entitled to a highly competitive salary. He didn‘t have enough
MONEY to buy that car.

E. BÀI TẬP THÊM:


101. _________for the money management 105. The president of Paterson Industrial
seminar will be forwarded to all the managers Solutions has signed a number of important
tomorrow. __________ this month
A. Invite A. contract
B. Invitations B. contracts
C. Inviting C. contracted
D. Invition
D. contracting
102. Even if residents in the area have shown
106. The ______of Mr. Alex Hartham was to
strong _________to the project, the city
initiate a merger with Culford International.
government may not be hesitant to proceed
A. intention
with it.
B. intended
A. resisting
C. intentional
B. resistant
D. intending
C. resisted
107. The chief financial officer of
D. resistance
Waterhouse InC. is expressing _________
103. The effects of affordable housing ____ in
concerning his predecessor's informative
rural areas will be discussed at the conference.
advice.
A. develops
A. appreciation
B. developing
B. appreciative
C. development
C. appreciating
D. developed
D. appreciates
104. When the____for this new safety policy
108. The _________ of a discount department
was under consideration by the managers, a
store was delayed by the protests of several
number of experts were asked for their
members of the community.
comments.
A. construct
A. proposing
B. constructive
B. propose
C. construction
C. proposes
D. constructed
D. proposal

UNIENGLISHCENTER | LESSON 6: GIỚI TỪ 136


109. Once you have your résumé with 115. You will have a full week at the resort
references and ____, please submit it to the of your………., departing on July 20 and
HR Department on the 3rd floor. arriving back in Tokyo on July 26.
A. qualified A. choose
B. qualifications B. choice
C. qualify C. choices
D. qualifying D. chose
110. _____ for reconstruction contracts has 116. The __________ of foods at Ricotta
been getting stiffer over the years since it Supermarkets makes it a popular choice
can generate enormously lucrative earnings. among consumers who enjoy preparing
A. competitively international cuisine.
B. competition A. various
C. competitive B. vary
D. competes C. variety
111. In order to brand your business, you‘d D. varied
better include your ……… in every letter you 117. The government sends teams of
send to customers, which helps remind them _________ to check the quality of the tap
of whom they do business with. water in both urban and rural counties.
A. signing A. inspected
B. signed B. inspectors
C. signature C. inspections
D. to sign D. inspect
112. Ms. Quong is a promising bookkeeper at 118. The __________ of our factory
our firm, as she completes ________ assigned equipment is vital for the high quality of our
to her with precision and quickness. manufactured goods.
A. working A. relied
B. worker B. reliable
C. work C. reliably
D. worked D. reliability
113. Much of the expected boost in revenue is 119. Sales __________ are encouraged to
related to a transaction tax which has yet to familiarize themselves with the location of
receive final……… from Congress. various merchandise in the department store.
A. approving A. representational
B. approved B. represent
C. approval C. representative
D. approves D. representatives
114. Mr. Lloyd has approved a _________ 120. Our firm received __________ from
for the construction of a new research facility numerous clients for our excellent work over
on the other side of the city. the past twenty years.
A. proposal A. compliment
B. proposed B. complimented
C. propose C. compliments
D. proposing D. complimentary

UNIENGLISHCENTER | LESSON 6: GIỚI TỪ 137


LESSON 1B: PRONOUNS
Đại từ được sử dụng để thay thế cho danh từ hay cụm danh từ để tránh lặp lại.
Những dạng Đại từ hay gặp trong bài thi TOEIC:
Đại từ chủ Đại từ tân ngữ
Đại từ phản thân Tính từ sở hữu Đại từ sở hữu
ngữ(S) (O)

Ngôi 1 số ít I Me Myself My Mine

Ngôi 2 số ít
You You Yourself/ Yourselves Your Yours
hoặc nhiều

Ngôi 1 số nhiều We Us Ourselves Our Ours

Ngôi 3 số nhiều They Them Themselves Their Theirs


He Him Himself His His
She Her Herself Her Hers
Ngôi ba số ít
It It Itself Its Its

1. Subjective ( Đại từ nhân xưng)


Đại từ nhân xưng đứng ở đầu câu hoặc trước động từ làm chủ ngữ trong câu.
Ví dụ:
- HE was given a chance to deliver an opening speech.
- I had a word with him yesterday and we decided to end our cooperation.
2. Objective ( Đại từ tân ngữ)
Đại từ giữ chức năng làm tân ngữ. Và sẽ có thể giữ chức năng tân ngữ trực tiếp hoặc gián tiếp,
đứng ngay phía sau ngoại động từ hoặc giới từ.
Víd ụ:
- We all loved HIM.
3. Đại từ sở hữu (Possessive pronouns) và tính từ sở hữu
Đại từ sở hữu được dùng để thay thế cho tính từ sở hữu và danh từ để tránh nhắc lại tín h từ sở
hữu + danh từ đã được dùng trước đó.
Ví dụ:
Which glass is yours. (yours = your glass)
Hoa‘s hat is blue and mine is white. (mine = my hat)
* Lưu ý: Tính từ sở hữu khác với đại từ sở hữu (My, your, his, her, its, our, your, their) là nó luôn
luôn đi với một danh từ. Nó đứng trước, bổ nghĩa và chỉ sự sở hữu danh từ đó chứ không thay thế
cho nó.
Còn đại từ sở hữu thì không có danh từ đứng sau nó như tính từ sở hữu.

UNIENGLISHCENTER | 138
Ví dụ:
This is my book. (Tính từ sở hữu + danh từ) This book is mine (Đại từ sở hữu)
4. Đại từ phản thân (Reflexive pronouns)
- Khi chủ ngữ và tân ngữ chỉ cùng 1 đối tượng ta dung đại từ phản thân thay thế tân ngữ
He cut himself.
Tom and his brother blamed themselves for the accident. He talked to himself.
The children have to look after themselves.
- Đại từ được sử dụng để nhấn mạnh danh từ . Khi đại từ phản thân nhấn mạnh danh từ trong câu
nó được đặt ngay sau danh từ đó.
Ví dụ:
The president himself gave him the gift. I spoke to the manager himself.
-By+ đại từ phản thân dùng để nhấn mạnh việc chủ ngữ tự làm lấy việc gì
She prepared the nine-course meal by herself. John washed the dishes by himself.

BÀI TẬP ĐẠI TỪ:


1. The two divisions have unveiled _____ 5. Workers are divised not to operate certain
plan to set aside millions of dollars to invest machines by ____.
in a very lucrative field. A. they
A. they
B. them
B. them
C. their
C. their
D. themselves
D. those
6. Indie film director Luke Steele will be in
2. Although the staff has grown, Mr. Lee
London for the premiere of _____ new movie.
continues to conduct all client meetings ____.
A. him
A. he
B. his
B. him
C. himself
C. his
D. he
D. himself
7. Clients wishing to cancel ___ appointment
3. Passengers on the aircrafl are asked to
must do so within 48 hours to avoid a penalty
secure _____ belongings during take off and
fee.
landing.
A. them
A. they
B. themselves
B. their
C. their
C. them
D. theirs
D. themselves
8. Ms. Woolner is hoping to change ____
4. Since she‘s running late for ____ meeting
daily routine to be more conductive to a
downtown, Ms. Joyce will have to catch a
healthy lifestyle.
taxi.
A. herself
A. she
B. her
B. her
C. she
C. hers
D. hers
D. herself

UNIENGLISHCENTER | 139
9. Most of the farmers in this area depend on 15. Susan Hardy will head the planing
local supermarkets and restaurants which buy committee and all members involved in the
fruits and vegetables from ____. new project will report back to ___
A. they she
B. them herself
C. theirs hers
D. themselves her
10. Mr. Whatley took Ms. Potvin‘s timetable, 16. Until Mr. Mato and Mrs. Kaori moved to
thinking it was ___. the newly renovated offices on the second
A. him floor, both of __ used temporary office space.
B. himself their
C. his them
D. he theirs
11. When interviewed, Susan explained that they
an inquisitive nature as a child made ___ 17. The successful candidates will be able to
interested in studying journalism. work effectively on ____ and as part of a
A. she team.
B. her A. they
C. hers B. their
D. herself C. their own
12. Ms. Goldberry marked ____ revision in D. themselves
red ink before returning the manuscript to the 18. Since Mr. Warren had finished writing his
author. monthly sales report ahead of schedule, he
A. she offered to help Ms. Ravera finish _____.
B. her A. hers
C. herself B. her
D. hers C. she
13. At this year‘s orientation, the general D. herself
manager _____ took the time to personally 19. Although the assistant manager will be
welcome all new recruits in her usual, affable transferred next week, the personnel office
manner. has not yet found a replacement for _____.
A. she A. she
B. her B. herself
C. hers C. her
D. herself D. hers
14. While Ms. Jamison‘s study focused on 20. Our skilled and experienced service
comsumer spending generally, ______ deals representatives will be able to resolve any
more specifically with purchasing trends problems you might encounter by ____.
among 18 to 24 years old. A. himself
A. I B. herself
B. my C. itself
C. me D. themselves
D. mine

UNIENGLISHCENTER | 140
LESSON 2: ADJECTIVES & ADVERBS
I.Hậu tố
 Nhận biết adj
Dưới đây là 8 hậu tố của TÍNH TỪ thường gặp trong bài thi TOEIC.

Suffix Example Meaning Suffix Example Meaning


FUL SuccessFUL Thành công IVE DecisIVE Tính quyết định
IC EconomIC Thuộc về kinh tế IAL SpecIAL Đặc biệt
ABLE ControllABL Có thể kiểm soát
LESS CareLESS Bất cẩn được
IBLE EligIBLE Có đủ khả năng

OUS VariOUS Đa dạng ENT EvidENT Rõ ràng

*Chú ý:
1. Những tính từ dễ gây nhầm lẫn
Economic economical
Sensible sensitive
Comparable comparative
Considerable considerate
Terrible terrific
Favorite favorable
Awful awesome
Successful successive
Exhausted exhaustive
Pleased pleasant

2. Tính từ với giới từ cố định trong TOEIC.


Be aware of Be likely to
Be eligible for Be ready to
Be able to + V_inf Be responsible for
Be capable of + V_ing Be subject to

UNIENGLISHCENTER | 141
Nhận biết adv
Trạng từ thường được hình thành từ TÍNH TỪ + LY.
Vídu: Extreme → Extremely
Lucky→ Luckily
Tuy nhiên, có một số trạng từ đặc biệt cần lưu ý:
 Một số vừa là tính từ vừa là trạng từ: Late Hard Near High Fast

II. ADVERBS
2. Some specialAdv
Highly/ high

Nearly/ Near

Hardly/ hard

Lately/ late

 Và một số từ kết thúc là đuôi LY nhưng lại là tính từ:

Friendly Timely Costly Daily Weekly Monthly Quarterly Lonely Elderly

UNIENGLISHCENTER | 142
3. Vị trí

Adj (bổ nghĩa cho N) Adv (bổ nghĩa cho V, adj, adv, câu)
1. Tính từ đứng trước danh từ bổ nghĩa 1. Trạng từ có thể đứng trước hoặc sau động từ để
cho danh từ. bổ nghĩa cho động từ.
 The DETAILED information The law requires that each item CLEARLY
display the warranty information.
2. Nếu xuất hiện giữa trợ động từ, tobe, model V
2. Tính từ đứng đằng sau động từ
hoặc O thì trạng từ đứng sau chúng.
to BE và 1 số động từ đặc biệt như
He was fired SHORTLY after getting into
LOOK, FEEL, GET, BECOME,
a heated debate with the manager about appropriate
MAKE, KEEP, SEEM, SOUND,
office etiquette.
REMAIN…..
3. Trạng từ đứng đầu câu bổ nghĩa cho cả câu.
 This operating system is not
FORTUNATELY, those difficult
COMPATIBLE with this model
customers are satisfied with the services we
computer.
offered.
HOPEFULLY, he will score the test the
3. Tính từ đứng đằng sau trạng từ để
way he expects.
được bổ nghĩa.
4. Trạng từ có thể đứng trước một trạng từ khác để

 The highly SUCCESSFUL fair was bổ nghĩa cho trạng từ đó.

attended by thousands of people. Members of the training department have been


working UNBELIEVABLY effectively over the last
 It is becoming progressively
3 weeks.
DIFFICULT for young people to
5. Trạng từ có thể đứng trước tính từ để bổ nghĩa
afford to invest in property.
cho tính từ đó.
He gave an EXCEPTIONALLY profound
4. Sau đại từ bất định
speech at the gathering.
 If you have anything interesting, let me
Safety regulations have recently been
see it.
reinforced to protect workers who work in
POTENTIALLY hazardous jobs.
6. Trạng từ đứng trước phân từ và hiện tại phân từ
She was BADLY injured in the accident
REGULARLY going out now may be very
dangerous

UNIENGLISHCENTER | 143
BÀI TẬP TÍNH TỪ
1. The conference‘s keynote speaker 6. Customers who purchase more than 500
addressed ….. impacts of digital technology dollars worth of items can request an HD
on the current music industry. cable box at no……….. charge.
A. variety additional
B. variably addition
C. variation adding
D. various additionally
2. Director Jun Iwata auditioned twenty actors
7. Construstion of the new shopping center
before making the ….. decision about whom
should be completed by the red of the year as
to cast in the main role.
long as economic conditions are …..
A. final
A. favor
B. finals
B. favors
C. finally
C. favorable
D. finalize
D. favorably
3. Because of a ….. increase in profits this
8. The research teams of Tyrell
quarter, Tyro Sportswear employees will
Telecommunications InC. possess …..
receive their first-ever year-end bouns.
knownledge of many of the product put forth
A. dramatically
by the competition.
B. dramatize
A. extend
C. dramatic
B. extends
D. drama
C. extensive
4. Doctor at Stephen Mallory Memorial
D. extent
Hospital claim thay cases of food poisoning
9. Arcadia Homes ….. for its commitment to
caused by improper storage are becoming
provide customers with quality furniture at
more …..
competitive prices.
A. frequented
A. is known
B. frequently
B. will know
C. frequent
C. to know
D. frequency
D. has known
5. Graduates from the Melrose College of
10. Registering and shelving new books and
Technology often make ….. contributions in
other media is now a ….. process since we are
several fields, including engineering,
down to seven librarians.
computer sciences, and astronomy.
A. length
A. value
B. lengthen
B. valuable
C. lengthy
C. valuably
D. lengthens
D. valuing

UNIENGLISHCENTER | 144
11. New owners of Century Regal‘s 16. Due to the approaching holiday season,
condiminiums are advided to become more Apline Board shop will now be ….. from 9
….. with the amenities and services available a.m to 5 p.m on weekends.
to residents. A. open
A. familiarity B. opens
B. familiarly C. openings
C. familiar D. to open
D. familiarize 17 .Like restaurants, caterers are subject to
12. There are some ….. jod ads in the ….. concerning safe food handing.
company newsletter in case any employee A. regulate
might know someone who can fill those B. regulations
positions. C. regulatory
A. updated D. regulating
B. update 18. As the systems ralated to …. cars change,
C. updating Standlane Internnational continually upgrades
D. update your software.
13. Underwood Enery Service InC. offers ….. A. electrically
benefits to full-time employees, including B. electricity
dental insurance. C. electrify
A. attraction D. electric
B. attractive 19. Our banquet rooms are capable of …..
C. attract events of all types and sizes.
D. attracted A. accommodate
14. If you need to hire ….. helf for your B. accommodated
growing business, our experts can assist with C. accommodating
the hiring process from start to finish D. accommodation
A. additions 20. The shutdown of General Electrics was
B. additionally ….. costly because the entire city could not
C. additional operate for more than three hours.
D. additions A. extremely
15. At yesterday‘s meeting, staff members B. extreme
were introduced to Peter McCabe, the C. extremes
recently ….. manager of the Dynasty Hotel in D. extremist
Washington D.C.
A. appointing
B. appointment
C. appoint
D. appointed

UNIENGLISHCENTER | 145
BÀI TẬP TRẠNG TỪ

1. The city council ….. approved the highway 7. Amateur theatrical performances by locals
extension project pending the results of an and some appearances by renowned poets
environmental review. were …. reviewed by journalists in the
A. conditions Eberfein County Daily.
B. conditioned A. positive
C. conditional B. positively
D. conditionally C. positivity
2. When handling large funds, bank D. positiveness
employees are required to check the 8. The CEO …. Appointed James Farley as
statements …. to make sure that everything is vice president of Arctic Mining
correct. Ventures Co.t.
A. careful A. formalize
B. carefully B. formally
C. more careful C. formal
D. carefulness D. formality
3. The two competing software program were 9. Shareholders votes that Deborah LaSaIIe is
determined to be …. effective in making …. suited for the position of chief operating
tiling taxes from home an easy task. officer given her expefience.
A. equal A. ideal
B. equally B. ideally
C. equaled C. idealized
D. equality D. ideality
4. Once Mr. Mirren …. completes his studies 10. A few vehicles were …. damaged by the
and necessary training, he may come and recent hailstorm and will have to be replaced
work for our corporation. immediately.
A. success A. severely
B. successful B. severe
C. successfully C. severeness
D. successes D. severity
5. Prattchet Co. has …. turned to making 11. After …. interviewing more than fifty
most of its products from recycled materials. candidates for the position of manager of the
A. increasingly Biotechnology Department, the committee
B. increasing chose Dr.Alan Liu.
C. increased A. carefully
D. increase B. to care
6. Ms. Maria Perreira is …. responsible for C. most careful
some of our company‘s most successful D. careful
marketing campaigns.
A. large
B. largely
C. largest
D. large

UNIENGLISHCENTER | 146
12. The Research and Development 16. You can …………. rebuild your credit
Department has come to the conclusion that by taking steps to repair your credit rating as
customers purchase kitchen appliances more soon as possible.
………… compared to twenty years ago. A. easy
A. attention B. easier
B. attentive C. easily
C. attentively D. easiness
D. attentiveness 17. The elegant hotel known for its fabulous
13. The price of organic foods is expected to amenities is……….located near the local
rise …. over the next several years because of airport.
an increase in demand. A. convenience
A. dramatize B. convenient
B. dramatically C. conveniently
C. dramatist D. conveniences
D. dramatic 18. Road and street signs must be …………
14. The new zoning laws should …. improve visible even in the distance in order for
the traffic congestion problem developing in drivers to know where they are heading.
the suburbs. A. clearness
A. great B. clear
B. greatness C. clearly
C. greatly D. clearest
D. greatest 19. It appears to be a ………… impossible
15. Mr. Garry Noxon in accounting was plan to charge fees to cars visiting the city
………… warned about having too many park on the weekend.
numerical errors in the budget report. A. nearing
A. repeat B. nears
B. repeatedly C. nearly
C. repeater D. neared
D. repetition 20. Please note that payment statements must
be addressed …………… to the recipients.
A. correct
B. correcting
C. correction
D. correctly

UNIENGLISHCENTER | 147
LESSON 3: TENSES

1. Simple Present Tense:


a) Cấu trúc:
(+) S + V (s/es) S + am / is / are +___
(-) S + do / does + not + V0 S + am / is / are + not +___
(?) (WH) Do / Does + S + V0? (WH) Am / Is / Are + S + ___?
b) Cách dùng chính:
 Hành động lặp đi lặp lại thường xuyên: always, often, usually, sometimes, seldom, rarely,
every day / week / month, one a week etc.
 Eg: Mary often gets up early in the morning.
 Sự thật hiển nhiên luôn đúng, chân lý.
 Eg: The sun rises in the East and sets in the West.
 Lịch trình tàu xe, thời khóa biểu, thời gian biểu (có thể xảy ra trong tương lai)
 Eg: The last train leaves at 4:00.
 Một luật lệ quy định hay sự việc cố hữu thường đi với các nhóm từ:
 Động từ chỉ giác quan: hear, see, smell, taste…
 Động từ chỉ tình cảm, cảm xúc: love, hate, like, feel like, fancy, dislike, detest, want,
wish…
 Động từ chỉ trạng thái, sở hữu: look, seem, appear, have, own, belong to, need…
 Động tử chỉ sinh hoạt trí tuệ: agree, understand, remember, know…
Bài tập:
A. Hoàn thành các câu sau dùng thì hiện tại đơn với động từ trong ngoặc.
1. (get) He always_______________to work at 8 o‘clock.
2. (work) They_____________as consultants.
3. (go) The manager_______________on business trip every month.
4. (review) When she_____________the report, she will bring it to you.
5. (start) My company‘s Christmas party____________at 9 tonight.
B. Lựa chọn đáp án đúng.
1. The sales meeting usually_______________at 5.
A. begins B. begin C. Beginning D. will begin
2. If he__________the team, we will start the project right away.
A. join B. joins C. joined D. will join

2. Present Continuous Tense:


a) Cấu trúc:
(+) S + am / is / are + V-ing
(-) S + am / is / are + not + V-ing
(?) (WH) Am / Is / Are + S + V-ing?
b) Cách dùng chính:
 Hành động xảy ra tại thời điểm nói thường đi với: now, right now, at the moment, at
present,…. Hay các động từ mệnh lệnh như: Look! See! Listen!...
 Eg: What are you doing at the moment? – I‘m writing a letter.
 Eg: Look! The bus is coming!
UNIENGLISHCENTER | 148
 Hành động mang tính tạm thời xung quanh thời điểm nói thường đi với: today, this week,
this month, these days…
 Eg: What‘s your daughter doing these days? – She is studying English at a foreign lauguage
center.
 Một dự định đã lên kế hoạch trong tương lai gần.
 Eg: What are you doing tonight? – I‘m going to the cinema with my father.

3. Present Perfect Tense:


a) Cấu trúc:
(+) S + have / has + V3/ed
(-) S + have / has + not + V3/ed
(?) (WH) Have / Has + S + V3/ed?

b) Cách dùng chính:


 Một việc xảy ra trong quá khứ chưa xác định rõ thời điểm và kết quả lưu đến hiện tại.
 Eg: I have lost the key. I can‘t open the door.
 Một việc xảy xa trong quá khứ kéo dài đến hiện tại dùng với ―since‖ hoặc ―for‖
 Eg: My friend has lived in HCMC since 1998.
 Một sự việc vừa mới xảy ra (thường có: just, recently, lately…)
 Eg: I have just finished my homework.
 Một trải nghiệm hay làm việc gì đó được bao nhiêu lần rồi.
 Eg: I have read this book 3 times.
 Trong cấu trúc:
 Be + the first / second…time + S + have / has + V3/ed
 Eg: This is the first time I have been to Paris.
 Be + the + so sánh nhất + N + S + have / has + V3/ed
 Eg: John is the laziest person I have ever met.
c) Trạng từ thường đi với thì HTHT: just, recently / lately, ever, never, yet, already, since, for, so
far / until now / up to now / up to the present, several times, for the last ten years…

4. Present Perfect Continuous Tense:


(+) S + have / has + been + V-ing
(-) S + have / has + not + been + V-ing
(?) (WH) Have / Has + S + been + V-ing?
Bài tập:
A. Chọn đáp án đúng trong ngoặc hoàn thành câu.
1. She (has just completed / have just completed) the work.
2. Oil prices (increase / have increased) a lot for the past 2 years.
3. They (has already finalized / have already finalized) the application.
4. We (visit / have visited) the city three times to attend the conferences.
5. Pam International (increases / has increased) its workforce by 10 percent over the last 2 years.
6. Since last summer, Daniel (has taken / took) three business trips to Europe.
B. Lựa chọn đáp án đúng.
1. Since last winter, they______financial difficulty.
A. has experienced B. experienced
UNIENGLISHCENTER | 149
C. have experienced D. experiencing
2. Mr. Jackson_______for the firm for 10 years.
A. has worked B. have worked C. worked D. working
3. The job fair_______place in Chicago last month.
A. takes B. take C. took D. has taken
4. Since last September, they_______a lot of profit.
A. make B. will make C. made D. have made
5. Mr. Ambrose, who is the chairman of PBN Steel Co.,______over $50,000 to charity every year
for the past decade.
A. contributed B. has contributed C. was contributing D. will contribute
6. Many residents in this city_______to install home security systems three months ago.
A. have decided B. decided C. are deciding D. decide
C. Chia động từ trong ngoặc.
1. (conduct) Last August, they___________a survey.
2. (try) They______________to complete the report since last Monday.
3. (quit) He___________.his job two months ago.
4. (work) Steve______________for this firm since 1998.
5. (finish) The team______________the investigation a few days ago.
6. (worry) Customers______________about the service for the past few years.

5. Simple Past Tense:


a) Cấu trúc:
(+) S + V2/ed S + was/were +___
(-) S + did + not + V0 S + was/were + not +___
(?) (WH) Did + S + V0? (WH) Was/Were + S +___?
b) Cách dùng chính: Diễn tả hành động xảy ra và hoàn tất trong quá khứ xác định rõ thời gian
thường có: yesterday, ago, last week / month / year, in the past, in 1990…
 Eg: My grandmother passed away in 1998.
Bài tập:
A. Hoàn thành những câu sau dùng thì quá khứ đơn với động từ trong ngoặc.
1. (work) I__________all night long yesterday.
2. (give) The president__________a speech yesterday.
3. (close) They__________the deal.
4. (like) The executive board__________my suggestion.
5. (write) I__________a letter of complaint last month.
6. (start) The film__________looking for workers three months ago.
B. Lựa chọn đáp án đúng.
1. My boss_____on a business trip last Friday.
A. go B. went C. goes D. going
2. Last week, he successfully_____the project.
A. finishes B. finish C. finished D. will finish
3. The conference room‘s alarm_____working last night because of damage caused by fire.
A. stopped B. stops C. stop D. is stopping
4. The mayor_____an address at the graduation ceremony of the nearby university every year.

UNIENGLISHCENTER | 150
A. gave B. gives C. will give D. is giving

6. Past Continuous Tense:


a) Cấu trúc:
(+) S + was/were + V-ing
(-) S + was/were + not + V-ing
(?) (WH) Was/Were + S + V-ing?
b) Cách dùng chính:
 Một hành động đang xảy ra tại một thời điểm xác định trong quá khứ:
 Eg: She was studying her lesson at 7 last night.
 Eg: What were you doing from 3pm to 6pm yesterday? – I was practicing English at that
time.
 Một hành động đang xảy ra trong quá khứ thì có hành động khác xen vào.
 Eg: He was sleeping when I came.
 Eg: While my mother was cooking dinner, the phone rang.
 Hai hành động diễn ra song song cùng 1 lúc trong quá khứ.
 Eg: While I was doing my homework, my brother was playing video games.
Bài tập:
A. Chọn đáp án đúng trong ngoặc hoàn thành câu.
1. I (am preparing / was preparing) a presentation at 9 last night.
2. The experts (discussing / are discussing) the matter in detail.
3. The technicians (was fixing / were fixing) the computers.
4. The committee (is reviewing / be reviewing) his application form now.
5. Some of the staff members (was participating / were participating) in the seminar.
B. Lựa chọn đáp án đúng.
1. Sam_____for a job, but there were not many.
A. is looking B. are looking C. was looking D. were looking
2. The executives_____an argument at the moment.
A. had B. are having C. were having D. having
3. Officials_____meeting with local residents over the next couple of weeks to discuss the
upcoming road construction work.
A. are B. were C. will D. have
4. Tens thousands of people_____the religious ceremony in Myers Park when the accident
occurred.
A. watch B. will watch C. would watch D. were watching

7. Past Perfect Tense:


a) Cấu trúc:
(+) S + had + V3/ed
(-) S + had + not + V3/ed
(?) (WH) Had + S +V3/ed?
b) Cách dùng chính: Khi 2 hành động cùng xảy ra trong quá khứ, hành động nào xảy ra trước
dùng QKHT, hành động nào xảy ra sau dùng QKĐ.
 Eg: We had had dinner before eight o‘clock last night.
 Eg: Lan had learned English before she came to England.
UNIENGLISHCENTER | 151
 Eg: By the time I left that school, I had taught there for ten years.
c) Dấu hiệu nhận biết: after, before, by the time, when, as, once…
Bài tập:
1. Scientists_____multiple areas of air pollution throughout the country before the law was passed.
A. are detecting B. had detected C. have detected D. will have detected
2. Pilots who_____commercial airliners for many years have secure pension plans.
A. was flying B. have flying C. flying D. have been flying

8. Past Perfect Continuous Tense:


(+) S + had + been + V-ing
(-) S + had + not + been + V-ing
(?) (WH) Had + S + been + V-ing?

9. Simple Future Tense:


a) Cấu trúc:
(+) S + will/shall + V0
(-) S + will/shall + not + V0
(?) (WH) Will/Shall + S + V0?
b) Cách dùng chính:
 Một quyết định tại thời điểm nói.
 Eg: I will call you tomorrow.
 Một dự đoán không có căn cứ.
 Eg: I think she will get married next month.
c) Dấu hiệu nhận biết: tomorrow, tonight, next week/month/year, some day, in the future…

10. Near Future Tense:


a) Cấu trúc:
(+) S + am/is/are + going to + V0
(-) S + am/is/are + not + going to + V0
(?) (WH) Am/Is/Are + going to + V0?
b) Cách dùng chính:
 Diễn đạt dự định hoặc kế hoạch trong tương lai.
 Eg: I have won $1000 dollars. I am going to buy a new TV.
 Diễn đạt dự đoán có căn cứ.
 Eg: Look at the dark clouds. It‘s going to rain.

11. Future Continuous Tense:


a) Cấu trúc:
(+) S + will/shall + be + V-ing
(-) S + will/shall + not + be +V-ing
(?) (WH) Will/Shall + S + be + V-ing?
b) Cách dùng chính:
 Một hành động xảy ra tại thời điểm xác định trong tương lai.
 Eg: At 12 o‘clock tomorrow, I will be having lunch with my friends.
 Một hành động đang xảy ra trong tương lai (TLTD) thì có hành động khác xen vào (HTĐ).
UNIENGLISHCENTER | 152
 Eg: They will be playing tennis when I come tomorrow.
Bài tập:
A. Chọn đáp án đúng trong ngoặc để hoàn thành câu.
1. The show (began / will begin) tomorrow.
2. The president (is going to retire / will retired) next year.
3. He (is going to join / are going to joining) the marketing team next month.
4. He (is going to visited / will visit) the L.A. branch in a few hours.
5. At this rate, Luke is going to (completed / complete) the project sometime next week.
6. The engineer (will drop / are going to dropping) by your office next Friday.
B. Lựa chọn đáp án đúng.
1. Helen is going to_____A&T Communications next week.
A. leave B. to leave C. left D. will leave
2. Hopefully, the package_____in a few days.
A. arrive B. arrived C. will arrive D. arriving

12. Future Perfect Tense:


a) Cấu trúc:
(+) S + will/shall + have + V3/ed
(-) S + will/shall + not + have + V3/ed
(?) (WH) Will/Shall + S + have + V3/ed?
b) Cách dùng chính:
 Hành động sẽ hoàn tất trước 1 thời điểm xác định hay trước 1 hành động khác trong tương
lai.
 Eg: It‘s now 7pm. I will have finished teaching this class by 8.30.
 Eg: By the time you come back, I will have written this letter.
c) Dấu hiệu nhận biết: By + time (by then, by the end of this week/month/year…)

13. Future Perfect Continuous Tense:


(+) S + will/shall + have + been + V-ing
(-) S + will/shall + not + have + been + V-ing
(?) (WH) Will/Shall + S + have + been + V-ing?
Bài tập:
A. Chọn đáp án đúng trong ngoặc hoàn thành câu.
1. Jim (has worked / had worked) for a consulting firm before he came here.
2. They didn‘t realize that somebody (had stolen / has stolen) their files.
3. As of next month, we (will work / will have worked) on this project for 10 months.
4. I didn‘t know that somebody (has broken / had broken) into my office.
5. By 2030, he (has been / will have been) in this business for 20 years.
B. Lựa chọn đáp án đúng.
1. As of next February, the accounting manager_____away for 2 years.
A. has been B. will have been C. had been D. been
2. Before they submitted the proposal, they_____a lot of research for it.
A. has done B. had done C. will have done D. done

UNIENGLISHCENTER | 153
BÀI TẬP THÌ

1. Even though assigned tasks me somewhat 6. Rather than the usual hosted dinner and
complex, we will have to complete them drinks, we will be____ tonight at the
before we ____ for the day. Mandarin Garden at 2025 Shattuok Ave, near
(A) leave the corner of Elm Street.
(B) leaving (A) dine
(C) will leave (B) dined
(D) have left (C) dining
2. By the time Clear Blaze Technology‘s (D) diner
word processing program goes on the market, 9. The Wallace Goldberg Committee of
software engineers ___ its remaining flaws Environmental Awareness unanimously_____
(A) will have corrected the technical team of Walpole Enterprises for
(B) had been correcting their annual award.
(C) are correcting (A) select
(D) will correct (B) selecting
3. Ms. Gamble, who ____ an influential (C) selected
literary critic in the 1980s, is now a (D) was selected
contributing writer to the National Gazette 10. A company picnic is____ for September,
newspaper. when most staff will be back from vacations,
(A) were although no date has yet been selected
(B) is (A) plan
(C) was (B) planning
(D) has been (C) planned
4. Ms. Chor relayed her concerns about the (D) plans
company‘s financial situation while she ____ 11. The falling price of Chinese-made
a meeting with the manager electronics last year _____ a number of
(A) having companies to plan innovations to their
(B) will have products.
(C) was having (A) causing
(D) has (B) causes
5. The assets of Marble Faun Publishing (C) has caused
Company _____ last quarter when one of (D) is caused
their main local distributors went out of 12. Soaring fuel prices have ____ to
business. manufactures seeking raw materials from less
(A) suffer remote suppliers.
(B) suffers (A) contributed
(C) suffering (B) contributions
(D) suffered (C) contribute
(D) contributor

UNIENGLISHCENTER | 154
13. Several staff heads____ a better 117. Maria Vega‘s keynote address at next
understanding of the importance of month‘s entrepreneurs Symbosium in
motivation and teamwork from the last Singapore____ by a short video
workshop (A) was preceded
(A) gaining (B) will be preceded
(B) will gain (C) would precede
(C) gained (D) were to precede
(D) were gained 18. Attendees will have a chance to watch
14. The technician____ repairs on the artist Marta Langdon as she ___ methods of
machinery, so production of the X220 will landscape painting at a workshop on Monday
resume when she has finished (A) demonstrating
(A) making (B) demonstrated
(B) had made (C) demonstrate
(C) will have been made (D) demonstrates
(D) has been making 19. By the time Ms. Valspar retires as
15. At the next meeting, the board of president of Movene Technologies, she ____
directors____ a new chairperson for the to increase the company‘s market share
forthcoming fiscal year. significantly.
(A) elected (A) manages
(B) elects (B) will have managed
(C) will elect (C) managed
(D) will be elected (D) has been managing
16. Beginning in August, free parking___ 20. The company____ working on the project
available to anyone taking the commuter train for the main office even before it chose the
from Green City land to build on
(A) were (A) will begin
(B) to be (B) had begun
(C) being (C) has begun
(D) will be (D) is beginning

UNIENGLISHCENTER | 155
LESSON 4: CÁC DẠNG THỨC CỦA ĐỘNG TỪ (BỊ ĐỘNG, V
KHUYẾT THIẾU)
Thể chủ động được sử dụng để nhấn mạnh tác nhân hoặc người thực hiện hành động. Trong khi thể
bị động lại được sử dụng để nhấn mạnh người nhận hành động đó, chứ không phải người thực hiện
hành động.
1. Điều kiện để có thể biến đổi 1 câu từ chủ động thành bị động:
V trong câu chủ động phải là Transitive Verb (Ngoại động từ: đòi hỏi có O theo sau)
2. Công thức chung của câu bị động: S+ (TRỢ ĐỘNG TỪ , TO BE)+ BE (chia
theo V chính)+V3/Ved
a. Xác định S, V, O và thì của V trong câu chủ động.
b. Lấy O trong câu chủ động làm S của câu bị động, giữ nguyên to be hay trợ động từ nếu có.
Lấy S trong câu chủ động đặt sau By trong câu bị động ( có thể bỏ qua)
c. Thêm To be vào sau S của câu bị động (To be phải chia theo thời của V chính trong
câu) chủ động và chia theo số của S trong câu bị động).
d. Biến đổi V chính trong câu chủ động thành PP2 (Past Participle) trong câu bị động và đặt
sau tobe.
Bảng công thức các thì ở thể bị động:
Tense Active Passive
Simple Present S+V+O S + am/is/are + PP.2
Present Continuous S + am/is/are + V-ing + O S + am/is/are + being + PP.2
Present Perfect S + has/have + PP.2 + O S + has/have + been + PP.2
Simple Past S + V-ed + O S + was/were + PP.2
Past Continuous S + was/were + V-ing + O S + was/were + being + PP.2
Past Perfect S + had + PP.2 + O S + had + been + PP.2
Simple Future S + will/shall + V + O S + will + be + PP.2
Future Continuous S + will/shall +be+Ving+ O S + will/shall +be+being+ PII

Future Perfect S + will/shall + have + PP.2 + O S + will + have + been + PP.2


Be + going to S + am/is/are + going to + V + O S + am/is/are + going to + be + PP.2
Model Verbs S + model verb + V + O S + model verb + be + PP.2
S + modal Verb + have +P2 S + modal Verb + have been +P2

UNIENGLISHCENTER | 156
Note:
-Có thể thêm BY+ người/ vật gây ra hành động nếu quan trọng và mang tính cụ thể
My aunt madethis rug= This rug was made( by my aunt)
-Một số cách diễn đạt đặc biệt

Be interested in: thích thú, Be dedicated to= be devoted to= Be accustomed to= be/ get
quan tâm be committed to= contribute to: used to: quen với
cống hiến, tận tụy
Be covered with: được bao Be concerned about= be confused Be related to= be involved in=
phủ bởi about be engaged in: liên quan, dính
líu vào
Be filled with: đổ đầy, lấp Be faced with: đối mặt với Be satisfied with= be pleased
đầy bởi with: hài lòng với
On/ Upon + Ving: khi It’s no use + Ving: vô ích Look forward to: mong đợi

Be busy (in) + Ving: bận bịu Be worth + Ving: đáng làm Object to: phản đối

Spend+ time/money+ Ving: Have difficulty/trouble/a


giành tiền, thời gian vào việc problem(in) + Ving: gặp khó khăn,
gì vấn đề rắc rối

Eg:
The customers are not satisfied with the quality of the product The management seems to be pleased with
the outcomes

UNIENGLISHCENTER | 157
MODAL VERBS
Chủ động: Modal Verbs + V
Bị động: Modal Verbs + be + PII

Modal Verbs Ví dụ
You can speak Spainish – Bạn có thể nói tiếng Tây Ban Nha
It can rain – Trời có thể mưa
Can (Có thể ở hiện tại, tương lai)
My brother could speak English when he was five – Anh trai tôi đã
Could (Có thể ở quá khứ)
có thể nói tiếng Anh khi anh ấy 5 tuổi.
You must get up earily in the morning – Bạn phải dậy sớm vào
buổi sáng.
Must/ Have to: Cần phải
You have to be here before 8 a.m – Bạn phải có mặt ở đây trước 8
giờ.
May (Có lẽ ở hiện tại) May I call her? Tôi có thể gọi cho cô ấy không?
Might (Có lẽ ở quá khứ) She might not be his house. – Cô ấy có lẽ không ở nhà anh ta nữa.
Will (Sẽ làm gì trong tương lai) Tomorrow will be sunny.
Would ( Giả định sẽ làm gì trong The problem will be taken care of by us
quá khứ) Did you buy sugar? Oh, sorry. I’ll go now
She was a child. She would be upset when hear this bad news.
Shall (Yêu cầu đề nghị dùng với I, Where shall we eat tonight?
we)
You should send this report by 8th September.
Should= Ought to (Nên làm gì) You ought to call her.
A decision should be made right away

UNIENGLISHCENTER | 158
BT THÊM:
1. East Abihay City is run ___ a mayor and 7. The contract stipulates that an employee is
six-member council who are elected for four _____ to do overtime during weeks that
years. presentations or special reports are being
A. Of prepared.
B. Among A. Requirement
C. By B. Requires
D. From C. Required
2. Even though construction ____ by D. Requisition
unexpected electrical storms, the new office 8. A replacement for the outgoing advertising
building was still finished in a timely manner. manager has not yet been ___, but the
A. Interrupts possibilities have been narrowed down to five
B. To interrupt applicants.
C. Was interrupted A. Chose
D. Be interrupted B. Choice
3. Because of the recent blizzard, student are C. Chosen
____ to submit their essays in the next class. D. Choose
A. Permitting 9. Each year, Gibbins Foods, a local food
B. Permitted manufacturer, ___ five thousand cans of
C. Permissible vegetables to the city‘s homeless shelters.
D. Permission A. Donates
4. The newest prototype ____ to give the B. To donate
engineers at Nova Enterprises a marked C. Donate
advantage over their competition. D. Are donated
A. Expects 10. No matter what the brand, computers
B. Is expected purchased at this shop will be ___ free of
C. Expected charge for a full year from the date of
D. Had expected purchase.
5. Questions about production concerns A. Service
should be ___ to Ms. Helen Yan. B. Servicing
A. Direction C. To service
B. Director D. Serviced
C. Directs 11. First-class seats are ___ with Internet
D. Directed hook-ups, global telephones and faxing
6. Once your payment has cleared, you ___ capabilities for all of your airborne
with a membership pass that is valid for 12 professional needs.
months from the purchase date. A. Equip
A. To issue B. Equipment
B. To be issused C. Equipped
C. Have issused D. Equipping
D. Will be issused

UNIENGLISHCENTER | 159
12. The manager is ___ to announce that 18. Membership at the exclusive sports club
Morgan & Co. will be giving stock options to ___ benefits ranging from the use of all sports
all interested employees. equipment to discounted rates at the club‘s
A. Please restaurant and bar.
B. Pleased A. Includes
C. Pleasant B. To include
D. Pleasing C. Have included
13. Even if Dr. Goldman does lead the study, D. Including
the level of his involvement in the project is 19. According to the Harton Fashion
___ to be determined. Chronicle many designers ___ green and
A. Yet brown fabrics for their autumn collections this
B. Rarely year.
C. Permanently A. To use
D. Besides B. Are using
14. After today‘s planning session is C. Had been used
completed, the members of each group will be D. Are used
___ a report of their team‘s objectives for the 20. Participants in the seminar were ___ to
next three years. make hotel reservations by the date indicated
A. Generation on the list of hotels.
B. Generate A. Advisable
C. Generated B. Advisory
D. Generating C. Advising
15. The Silvau Division is now ___ a full line D. Advised
of steel products at a new modern facility just 21. Access to the computer terminals ___
outside the city. during evening hours, so it is necessary for us
A. Manufacturer to finish the work during the day.
B. Being manufactured A. Restrict
C. Manufactured B. To restrict
D. Manufacturing C. Is restricted
16. Harwood Department Store will be ___ D. Has restricted
later this weekend to allow our shoppers to 22. The first electric cars were ___
take advantage of our end –of-season impractical in that they could not be used to
clearance sale. travel long distances.
A. Close A. Considerate
B. Closes B. Considered
C. To close C. Considering
D. Closing D. Considerable
17. Nancy Freeman is ambitious and strategic 23. Please mark the appropriate box to
and, therefore, is ___ the top candidate to take indicate if the person named above ___ any of
over as manager of public relations. the certificates listed below.
A. Consider A. Holding
B. Considerable B. Held
C. Considerate C. Holds
D. Considered D. Hold
UNIENGLISHCENTER | 160
LESSON 5: LIÊN TỪ
Liên từ là từ dùng để nối hai từ, cụm từ hay mệnh đề trong một câu
1. Liên từ đẳng lập (and, but/ yet, or, nor, so);
For = Because: giữa câu, sau dấu phẩy
And = và
Nor = No
But = Nhưng/ except for
Or = hoặc
Yet = nhưng
So = Vì vậy
2. Liên từ kết hợp:
both A and B (vừa A vừa B)
either A or B (hoặc A hoặc B)
neither A nor B (không A cũng không B)
not only . . . but also… (không những…mà còn…),
whether . . . or… (liệu rằng…hay…)
as well as (cũng như)

Check- up

3. Liên từ mở đầu mệnh đề danh từ (that, if, whether)


- Trong tiếng Anh, có những mệnh đề đứng ở vị trí và đóng chức năng của một danh từ.
Đó chính là mệnh đề danh từ.
UNIENGLISHCENTER | 161
- Mệnh đề này thường bắt đầu bằng if, whether và các từ để hỏi như what, why,
when, where, …và từ that, the fact that
+ Why she is absent is nothing to me. ( mệnh đề danh từ làm chủ ngữ)

+ I don‘t know what she wants. ( Mệnh đề danh từ làm tân ngữ

+ They think that they are wrong. ( Mệnh đề danh từ làm tân ngữ)

+ Can you show me where they are now? ( Bạn có thể cho tôi biết giờ này họ đang ở đâu không?)

Check-up

4. Liên từ mang nghĩa ‘ vì, bởi vì’


Because of / Due to/ Owing to/ thanks to/ as a result of + V-ing / N
Ex: We didn‘t go out due to the cold weather.
They are here because of us.
Because / As / Since/ Now that +Clause
Ex: They can‘t go out because / as / since it is very cold outside.

5. Liên từ mang nghĩa ‘ dù, mặc dù’


Although/ even though/ though/ even if + Clause
Regardless of/ Despite/ in spite of + Ving/ N
While/ whereas/ on the other hand
Although he was lazy, he passed the test.
UNIENGLISHCENTER | 162
Despite studying hard, he failed the final test.
We had a picnic in spite of the bad weather.
I am tall, whereas my wife is quite short
6. Liên từ chỉ mục đích ‘ để làm gì’
to / in order to / so as to - Vbare…
He went to France to study French.
He does morning exercises in order to improve his health.
so that / in order that + S + will / can / would / could …+ Vbare
I‘ll try my best to study English so that I can find a better job.
Check up

7. Liên từ chỉ điều kiện ( if/ as long as/ provided= providing that)
 Câu điều kiện loại 1
Diễn tả sự việc có thể xảy ra ở hiện tại hoặc tương lai.

Công thức:

If + clause (hiện tại đơn), S + will/can/may/must… + V (+ O)


Ví dụ:

If it is sunny, I will go fishing.(Nếu trời nắng tốt, tôi sẽ đi câu cá.)

UNIENGLISHCENTER | 163
If you want to lose weight, you should do some exercise.(Nếu bạn muốn giảm cân, bạn nên tập thể
dục.)
 Câu điều kiện loại 2
Diễn tả sự việc không có thật hoặc không thể xảy ra ở hiện tại hoặc tương lai.
Công thức: If + clause (quá khứ đơn), S + would/might/could … + V (+ O)
Ví dụ: If I were you, I would tell the truth.(Nếu tôi là bạn, tôi sẽ nói sự thật.)=> thực sự tôi
không thể là bạn.
If I had a million USD, I might buy that car.(Nếu tôi có một triệu đô la, tôi có thể mua chiếc xe đó.)
=> hiện tại tôi không có số tiền như thế.
 Câu điều kiện loại 3
Diễn tả sự việc không có thật ở quá khứ.
Công thức: If + clause (quá khứ hoàn thành), S + would/could/might/….+ have + V3/ed
(+ O)
Ví dụ:
If I had known the answer, I would have told her.(Nếu tôi biết câu trả lời, tôi đã nói với cô ấy.)
If I hadn‘t been absent yesterday, I could have met him. (Nếu hôm qua tôi không vắng mặt, tôi đã
gặp anh ấy)
 Note 1: Unless = If….not
Unless (trừ phi…nếu không) thay thế cho if….not trong câu điều kiện loại 1.
Mệnh đề theo sau unless cần chuyển về dạng khẳng định.
Ví dụ :
You will not pass the examinations if you don‘t study hard.
= You will not pass the examinations unless you study hard.(Con sẽ không đậu kì thi trừ khi con
học hành chăm chỉ.)

UNIENGLISHCENTER | 164
 Note 2: Đảo ngữ trong câu điều kiện
Đảo ngữ là hình thức đảo ngược vị trí thông thường để nhấn mạnh một thành phần hay ý nào đó
trong câu
Loại điều kiện Cấu trúc thường Cấu trúc đảo ngữ

Loại 1: Tình huống If + S + should+ V (+ O), S + Should + S + V (+ O), S +


có thể xảy ra ở hiện will/should/can… + V (+ O) will/should/can… + V (+ O)
tại hoặc tương lai E.g.: If you like her, you can E.g.: Should you like her, you
nhưng khả năng xảy invite her to come to your can invite her to come to your
ra thấp Đảo should party. party.
lên thay cho if
Loại 2: Tình huống giả If + S + were+……, S + Were + S +……, S +
định, không thể xảy would/might/could… + V (+ would/might/could… + V (+
ra hoặc không có thực O) E.g.: O) E.g.:
ở hiện tại - If I were you, I would not - Were I you, I would not do
(đảo were lên thay cho do such a rude thing. such a rude thing.
if) - If she got up late, she could - Were she to get up late, she could
not catch the train. not catch the train.
Loại 3: Tình huống If + S + had + V3/ed (+ O), S Had + S + V3/ed (+ O), S +
trái thực tế trong + would/might/could… + would/might/could… + have +
quá khứ have + V3/ed (+ O) V3/ed (+ O)
(Đảo had lên thay E.g.: If it hadn‘t been for your E.g.: Had it not been for your help,
cho if) help, I wouldn‘t have succeeded.
I wouldn‘t have succeeded.

I/ Câu điều kiện loại 1, 2, 3


a. If the budget had not been tight, we would fix/ would have fixed the facilities
b. If he gets/ got the job, I will be very surprised
c. If she had finished the report on time, she can/could have attended the meeting
d. If the company offered/had offered some incentives, its employees would have been happier
e. If we have/had more time, we could finish the project successfully
f. If we had a fax machine at home, we will send/ would send him the document
g. If Mr. Romeo works/ worked harder, he could become a team leader
II/ Đảo ngữ trong câu điều kiện

UNIENGLISHCENTER | 165
a. Were/if he nice to his coworkers, they would enjoy working with him
b. Were I the boss, I will/would give Mr. Brown a raise.
c. Had/Have the project not been successful, she wouldn‘t have been the best employee of the year
d. Had he had a degree in business, he could have gotten/ had gotten the marketing position.
8. Liên từ chỉ thời gian
+ When /While/ as ( khi mà) = During + N (sự kiện, tình huống)
 When he comes, I will make a cake for him.
 I am reading book while they are playing.
+ As soon as/ once/ Just as (ngay khi mà)
 Ví dụ: As soon as Marry knew the truth, She called me.
 Ví dụ: Just as the girl entered the room, everyone looked at her.
+ Before (trước khi)
Ví dụ: She often watches TV before she goes to bed .
+ After (sau khi)
 He went out after he had finished the housework.
+ Till/ until ( cho đến 1 thời điểm nào đó và sẽ kết thúc ngay tại đó) ( open, valid, last)
 Ví dụ: I‘ll waite untill/ till he comes back.
+ Whenever ( bất cứ khi nào)
 Ví dụ: I‘ll come whenever you need me.
Khi chỉ 1 việc sẽ xảy ra trong tương lai ta thường dùng hiện tại đơn trong mệnh đề trạng ngữ
chỉ thời gian, và tương lai đơn trong mệnh đề chính.
+ By the time + Clause ( hiện tại đơn), Clause( tương lai hoàn thành)-> hành động ở hiện tại
 Ví dụ: By the time she comes, the shop will have closed.
+ By the time + Clause ( quá khứ đơn), Clause( quá khứ hoàn thành)-> hành động ở quá khứ
 By the time I got there, everyone had left.
+ Since ( kể từ khi)= ever since
 Ví dụ: I haven‘t met them since they left.
Chú ý: ” since ” thường được dùng trong thì hiện tại hoàn thành.
9. Một số liên từ khác.
Therefore, consequently ( vì thế vì vậy)
I have been getting used to job. Therefore, it is quite difficult to quit ỉt right now
However, nevertheless (nhưng, tuy nhiên)
She refused to take that position. However, the manager kept asking her to do it

UNIENGLISHCENTER | 166
Otherwise ( nếu không thì)
Make a note some of the questions you want to ask. Otherwise, you will forget some of them.
Besides/ moreover/ furthermore/ in addition
You can make cookies as long as you want. Besides, you can take them home if you want.

BÀI TẬP THÊM:


1. You are to submit ___ your medical 6. Alvinson Clothiers‘ branch in Milan has
records and registration form to the been open since last August ___ is quickly
admisnistrative office. becoming our busiest location.
A. as well as A. And
B. as soon as B. Also
C. either C. Or
D. both D. Either
2. Ms. Fenwick not only composed the report 7. All employees are required to attend ___
___ produced the visual aids which were so the morning and the afternoon sessions of the
useful during the meeting. training workshop on Friday.
A. But A. When
B. However B. Both
C. Aslo C. Which
D. And D. Some
3. ______ Mr. Fowler or Ms. Bloch will have 8. We seek to hire employees who either have
to travel to London for the symposium on experience in the field ___ have completed an
textile manufacturing. HVAC training course at an approved
A. Both institution.
B. Each A. If
C. Either B. But
D. Neither C. So
4. Furniture from our supply centers can only D. Or
be ordered ___ directly over the phone or 9. The Sunderland building has ___ a garage
online. nor a parking lot, so tenants must find spaces
A. And to leave their vehicles by themselves.
B. either A. Every
C. both B. Neither
D. yet C. All
5. Orders for specialty flower arrangement D. Either
from thyme Orchid can be placed ___ the 10. All the forms you submit must be
phone or online. postmarked no later than the specified date
A. If ___ they will be considered untimely.
B. In A. Or else
C. Either B. Besides
D. On C. On account of
D. Either

UNIENGLISHCENTER | 167
11. Two forms of currently valid 15. This sewing workshop is ideal for
identification, including ___ a driver‘s license beginner
or a passport, must be presented. tailors ------- professionals who would like
A. Neither to refresh their knowledge.
B. Whether A. but
C. Either B. either
D. Both C. as well as
12. Shoppers may bring a total of five D. additionally
clothing items ___ two accessories into the 16. ------- the rent increase is less than 2
fitting room at a time percent, Selwin Electrical Supply will
A. While continue to lease the space.
A. As long as
B. So
B. Along with
C. Though C. Not only
D. And D. Otherwise
13. ------- he is usually quiet around the 17. The quality assurance department
office,Mr. Heineman is known for his needs to hire additional staff -------
witty and engaging public speeches. production has increased by 50 percent.
A. Although A. even though
B. since
B. Otherwise
C. because of
C. Despite D. therefore
D. Instead 18. Our most recent survey was sent to
14. Our most recent survey was sent to clients just last week, ------- it is too
clients just last week, ------- it is too soon to send another one.
soon to send another one. A. when
A. when B. since
C. so
B. since
D. finally
C. so 19. The annual report has been posted
D. finally online,------- the director's office has
not yet received a printed copy.
A. but
B. why
C. with
D. once
20. ------- it is relatively small, the fitness
center at Ginder Apartments is very
popular with residents.
A. Reasoning
B. Essentially
C. Although
D. Throughout

UNIENGLISHCENTER | 168
LESSON 6: GIỚI TỪ
Đứng trước danh từ để liên kết danh từ với các thành phần khác trong câu

Thời gian: 1 thời điểm At 5 o‘ clock, at lunch time, at midnight, at


cụ thể present,.... At Christmas, at weekends,...
At
At the theatre, at the window, at the cinema, at the post
Nơi chốn: 1 địa điểm cụ thể
office
Thời gian: Chỉ 1 ngày On July 1, on Sunday,...
On Nơi chốn: tiếp xúc, nằm trên
On the ceiling, on the wall, on the table,....
bề mặt
Thời gian: dài hơn ngày
hoặc ngắn hơn ngày(
In July, in the evening, in 10 minutes,....
buổi,
tháng, năm, mùa, thế kỉ,...)
In
Nơi chốn: trong 1
In the room, in the drawer, in the building,...
không gian kín
Trong 1 địa điểm lớn
In New York, in China,...
như thành phố, đất nước
Chỉ 1 khoảng thời gian ( từ
for The survey conducted lasted for 3 weeks
quá khứ đến hiện tại
Chỉ 1 khoảng thời gian
in trong I will be at home in 3 minutes
tương lai (sau…)
Thời gian: Trong vòng,
It is imperative that the company lower its expenses within
không vượt quá 1
a month
khoảng thời gian nào đó
within
Khoảng cách: Trong vòng,
Stay within 3 kilometes
không vượt quá 1
Keep the boy within the room until 6p.m
khoảng nào đó
Thời gian: xuyên suốt từ
Throughout khi bắt đầu đến cuối 1 There is a 20% chance of heavy rain throughout the day
khoảng

UNIENGLISHCENTER | 169
thời gian nào đó

Nơi chốn: trên khắp từ


điểm bắt đầu đến cuối 1 Throughout the world, throughout the province
nơi nào đó
Thời gian: từ khi nào đến
From 7a.m to 7p.m
From...to khi nào
Nơi chốn: từ đâu đến đâu From the school to the post office
Thời gian: trước hay
muộn nhất cho đến thời
Finish by tomorrow
điểm nào đó
By (complete, finish, submit)
Nơi chốn: bên cạnh By the window
Phương tiện, cách thức By car, by hand, by fax
= through= with Through internet
Without Màkhông có She went out without any money

1 số giới từ khác hay xuất hiện trong toeic


Between (giữa 2 người, vật)
Among (giữa 3 người, 1 nhóm trở lên)
Come/ go into effect: có hiệu lực
Rely/depend on: phụ thuộc vào
In accordance with: để phù hợp với
Behind schedule/ on schedule: chậm tiến độ, đúng tiến độ
Be under way : đang được tiến hành
Prior to: trước= before
Apply for/ apply to: nộp đơn xin việc/ áp cái gì vào cái gì
Come to an end: kết thúc, chấm dứt
According to: theo như
In addition to= besides: thêm vào, ngoài ra
Concentrate on/ focus on: tập trung vào
Interfere with: can thiệp, xen vào
On behalf of: thay mặt cho

UNIENGLISHCENTER | 170
In need of: cần
Get through: hoàn thành, làm trọn
Provide sb with st= offer/ give sb st
Prevent/ prohibit/ discourage A from B: ngăn cản ai làm gì
Comply with= abide by= adhere to: tuân thủ theo
Demand for: nhu cầu về …
By means of: bằng cách
Barring any further delays: Trừ phi có trì hoãn thêm
Questions concerning/ regarding the report: Câu hỏi có liên quan đến bản báo cáo
Following: theo sau
Including: bao gồm
Excluding: ngoại trừ
Decrease/ increase/ rise in oil prices: tăng, giảm về cái gì
Decrease/ increase by 20%: tăng, giảm bao nhiêu
Decrease/ increase to 90%: tăng giảm tới mức…
In celebration of: để kỉ niệm
At no cost/ at no charge/ free of charge : miễn phí
On one‘s way to: trên đường tới…
Under warranty: đang được bảo hành
Upon request: khi có yêu cầu
Above one‘s expectation: trên cả mong đợi
In excess of vượt quá
At the latest: muộn nhất
No later than: không muộn hơn
In writing: bằng văn bản
Be noted for/ be famous for/ be known for: nổi tiếng về

UNIENGLISHCENTER | 171
BT THÊM:
1. ------- representatives of Light 7. The director of the Yeon Park Clinic will
CloudAirlines, flights are rarely deliver a speech ------- tonight's
overbooked. benefit dinner.
A. As opposed to A. except
B. In addition to B. before
C. According to C. as
D. Prior to D. onto
2. Aiko Arts plans to host a reception ------- 8. The economic development summit will be
the artist, Remco Koeman. held ------- the Xi'an Trade Tower on
A. except September
B. for A. to
C. off B. at
D. into C. down
3. All posters and flyers must be removed D. of
from the bulletin board ------- 48 hours after 9. ...........for approved visitors, no one from
theevent has occurred. outside the company is allowed to gain
A. within entry to the factory floor.
B. among A. While
C. unless B. So
D. while C. Except
4. Any employees and their family D. As
members wishing to visit the exhibition 10. I need you to find a solution..........the
may do so ...........no cost. problem before the budget meeting
A. at begins tomorrow.
B. by A. over
C. over B. about
D. from C. to
5. The award-winning film UndetWater D. out
Secrets 11. Information ------- the artwork in the
promotes awareness------- ocean pollution lobby is available at the reception desk.
and its effects on our planet. A. across
A. of B. about
B. to C. upon
C. from D. except
D. with 12. Mr. Lincoln promised to submit his
6. ------- construction in progress at the estimate for the cost of the
old town hall building, tourist groups project...........the end of next month.
will not be allowed to enter the site. A. by
A. Now that B. up
B. While C. of
C. Even if D. among
D. Due to

UNIENGLISHCENTER | 172
13. The negotiations are in their final stage 17. ...............the national holiday, there will
and should end........5 P.M. be
A. prior no mail service tomorrow.
B. ahead A. According to
C. early B. Because of
D. before C. However
14. Passengers must be at the gate ----25 D. Therefore
minutes prior to boarding their 18. ------- the popularity of the BPT39
flight. wireless speaker, production will be
A. at least increased fivefold starting next month.
B. as much A. On behalf of
C. in case B. Whether
D. by then C. Moreover
15. The prime minister greeted foreign D. As a result of
diplomats ------- a luncheon today 19. Videos of Korean pop music have
inLondon. become very popular -------
A. at adolescents worldwide.
B. had A. including
C. such B. whereas
D. where C. among
16. Russet Software announced in a press D. within
release that it is Russet Software 20. Ms. Murata requests that this month's
planning to expand -------- India. sales totals be submitted ------- the
A. at end of the day.
B. about A. within
C. into B. if
D. of C. that
D. by

UNIENGLISHCENTER | 173
LESSON 7 - RELATIVE CLAUSE
I. ĐỊNH NGHĨA
- Mệnh đề quan hệ (relative clause) là một mệnh đề phụ, có chức năng như một tính từ được dùng
để giải thích rõ hơn hoặc bổ nghĩa cho danh từ đứng trước nó.
- Mệnh đề quan hệ được bắt đầu bằng một đại từ quan hệ (relative pronouns: WHO, WHOM,
WHICH, WHOSE, THAT) hoặc trạng từ quan hệ (relative adverb: WHEN, WHERE, WHY)
Eg: The woman who lives next door is kind
II. ĐẠI TỪ QUAN HỆ VÀ TRẠNG TỪ QUAN HỆ:
1. ĐẠI TỪ QUAN HỆ

Đại từ quan hệ Cách sử dụng Ví dụ

Thay thế cho người, làm chủ An architect is a person who designs
Who
từ trong MĐQH buildings.
The man whom/ who I want to see wasn‘t
here.
Thay thế cho người, làm tân
Whom/ Who
ngữ trong MĐQH.
The woman to whom you’re talking is my
boss.

Chỉ sở hữu của người, vật, thay I have a friend whose father/ the father of
Whose
thế cho tính từ sở hữu ( his- , whom is a famous restaurant owner.
( of which,
her- , its- I have a table whose leg/ the leg of which has
of whom)
, their- , our- , my- , -‘s ) been broken.

The book which is on the table is mine.


Thay thế cho con vật, đồ vật. có
Which thể làm chủ ngữ hoặc tân ngữ
Fill out this application form, which I gave to
trong MĐQH
you

Dùng thay cho who, whom,


which trong mệnh đề quan hệ
That xác định( không có dấu phẩy). I need to meet the boy that is my friend’s son
Có thể làm chủ ngữ hoặc tân
ngữ trong MĐQH

UNIENGLISHCENTER | 174
Chú ý: What có thể thay thế cho the thing which/ the thing that
Eg: The accident which/ what happened last night can have a great impact on our industry
Eg: This is the manual which/ what I have been looking for

2. TRẠNG TỪ QUAN HỆ: Trạng từ quan hệ được sử dụng thay cho một đại từ quan hệ và
giới từ.

Thay thế cho cụm từ chỉ thời


When (in/on/at gian: Do you remember the day when/ on
which) then, at that time, on that which we fisrt met ?
day….
I like to live in a country where/in
Where (in/at /on Thay thế cho cụm từ chỉ
which human rights are taken
which) nơi chốn trong MĐQH
seriously
Thay thế cho cụm trạng từ chỉ Tell me the reason why/ for which
Why (for which)
lído : for that reason you are late for yesterday’s meeting?

UNIENGLISHCENTER | 175
III. DẠNG RÚT GỌN CỦA MỆNH ĐỀ QUAN HỆ
1. Dùng Ving : khi mệnh đề quan hệ dạng chủ động, thì ta có thể bỏ đại từ quan hệ và
động từ sau nó phải chia dạng Ving
Eg: The woman who is sitting next to you is my immediate supervisor.
→ The woman sitting next to you is my immediate
supervisor. (Người phụ nữ đang ngồi cạnh bạn là quản lý trực
tiếp của tôi.)
Eg: Do you know the man who won the Best Employee Award?
→ Do you know the man winning the Best Employee Award?
(Anh có biết người đàn ông giành giải thưởng Nhân viên xuất sắc là ai không?)
2. Dùng quá khứ phân từ: khi mệnh đề quan hệ ở dạng bị động thì ta có thể bỏ đại từ
và động từ sau nó phải chia ở Phân từ II.
Eg: The film which was made by him won many prizes.
→ The film made by him won many prizes.
(Bộ phim của ông ta đạt nhiều giải thưởng).
Eg: The building which was built many years ago collapsed.
→ The building built many years ago collapsed.
(Tòa nhà được xây dựng nhiều năm về trước đã
đổ). Notes:
- All/both/several/most/ some/ any/ few/neither/either......+of + which/whom

UNIENGLISHCENTER | 176
Eg: The program enrolls 20 students. All of the students are
women
 The program enrolls 20 students, all of whom are women.
- Đại từ quan hệ làm tân ngữ và không có giới từ phía trước (who, whom, which,
that)-> lược bỏ đại từ quan hệ
Eg: The girl (whom/who) I met yesterday is my best friend
Eg: He took the medicine (which) the doctor prescribed

BÀI TẬP THÊM:


1. Customers ________ wish to return 5. The campaigns that ________ asssigned to
merchandise within 7 days of purchase must the broadcasting division have been scrapped
present the valid receipt to the store. due to lack of support.
(A) who (A) was
(B) when (B) were
(C) what (C) be
(D) whom (D) has
2. You may return for full credit any 6. Karen Solden will replace Harold Walker,
merchandise with ________ you are not ________ is being promoted to a higher
satisfied. position in the company.
(A) who (A) he
(B) what (B) who
(C) which (C) his
(D) whose (D) whose
3. For those ________ are determined to 7. An investment ________ gradually
attend the seminar, tickets are available at the increases in value over the years can be more
Human Resources Department. profitable than more short-term alternatives.
(A) who (A) that
(B) whose (B) they
(C) what (C) what
(D) their (D) when
4. The main artists ________ paintings are 8. Attendance is mandatory for the staff
being sold at the Garrett Gallery are Elaine meeting ________ for next Tuesday in the
Crick, Jason Chambers and Sylvia Web. council hall.
(A) who (A) has been scheduled
(B) whose (B) schedule
(C) which (C) will schedule
(D) whom (D) scheduled

UNIENGLISHCENTER | 177
9. Anyone ________ experiences 15. Mr. Thompson collected a list of
complications with the new software is employees ________ were unable to attend
encouraged to bring this matter to Mr. the offical company picnic.
Gruber‘s attention in room 210. (A) whose
(A) who (B) who
(B) which (C) whom
(C) whom (D) whoever
(D) whose 16. The package ________ was sent to Mr.
10. Customers ____ purchase their orders Jones was returned to us when the mail
from the website of Westminster Books carrier could not locate the address on the
receive an automatic discount of at least 1%. shipping label.
(A) who (A) when
(B) whom (B) who
(C) whomever (C) what
(D) whose (D) that
11. Ella Portofino, ________ is known for 17. The incoming chief executive knows the
her motivational speeches, will be the guest actions ________ need to be done to turn
speaker at Leadership Conference in June. around the company‘s performance
(A) each (A) why
(B) which (B) that
(C) who (C) who
(D) this (D) what
12. Conference attendees _______ confirm 18. The names of the department heads to
their attendance early will be eligible for pre- ________ the monthly reports should be sent
assigned seating. are located on the last page of the manual.
(A) whose (A) whoever
(B) what (B) whom
(C) who (C) what
(D) whom (D) where
13. Anyone who ________ a new stove by 19. The expert ________ spoke at last
CTX LtD., this weekend receives a free four- month‘s seminar on environmental
piece set of stainless-steel saucepans. awareness has been nominated for a Flagler
(A) purchase Price.
(B) purchaser (A) whoever
(C) purchases (B) which
(D) purchasing (C) whenever
14. Changes in our organizational structure (D) who
have created ways for employees to explore 20. Be aware that candidates ________
opportunities ________ were previously not applications are incomplete will not be
available. considered for an interview
(A) what (A) who
(B) those (B) whose
(C) that (C) their
(D) there (D) they
UNIENGLISHCENTER | 178
LESSON 8 – PHÂN TÍCH CÁC DẠNG ĐỌC HIỂU PART 7
DANG 1: EMAILS/ LETTERS
Questions 1-3 refer to the following letter. :

Hollywood Travel Plus


2498 Broadway
Hollywood, CA
July 14, 2008
Dear Sir or Madam,
I saw your company's billboard advertisement promoting a package trip to Costa Rica. I am
interested in learning more about this offer.
The ad listed the price for the package, including airfare and accommodation for 6 nights, as
$1,180. This seems like a very cheap price to me, and I'm wondering how it is possible. What
type of hotel accommodation is included in the package, and where is it located? I am hoping to
stay at a five-star hotel somewhere on the Pacific coast of Costa Rica. If the advertised package
does not include such a hotel, are you offering other packages that do?
I would appreciate if you could contact me soon to answer my questions, as I am planning a
vacation for mid-September. You can call me at my cellphone at 714-555-24488
Most thankfully,
Vicky Siebens

1. Why was the letter written?


(A) To describe a recent travel experience
(B) To make hotel reservations in a foreign country
(C) To request information about a package vacation ned
(D) To complain about a billboard advertisement
2. What feature is Ms. Siebens looking for about a hotel?
(A) A friendly atmosphere .
(B) An affordable price
(C) A high quality rating
(D) A downtown location
3. When does Ms. Siebens want to take trip?
(A) In one week
(B) In two weeks
(C) In one month
(D) In two months

UNIENGLISHCENTER | 179
Questions 4-5 refer to the following email.

From: Lared Petrovski, Office Manager <lared@naracore.com>


To: Dan Vickerman, Administration Assistant <vickerman@naracore.com>
Re: Tomorrow's conference preparations

Hi Dan,

I need you to help me set up the conference room for tomorrow's event. We have to
arrange the tables, set up the audio-visual equipment, and prepare the afternoon tea, The
conference starts at noon, so you should start helping me at around 9:30 a.m. If it's just
the two of us, I guess it will take a couple of hours to get everything done. But if there's
anyone else in the administration department who's available to help, please bring them
along. I'd like to get as many people involved in the preparations as possible, so that we
can be ready by 11 a.m.

Thanks,
Lared Petrovski

4, When is the event scheduled to begin?


(A) 9:30
(B) 11:00
(C) 11:30
(D) 12:00

5. What does Mr. Petrovski ask Dan Vickerman to do?


(A) Ask other people to volunteer
(B) Buy some audio-visual equipment
{C) Call the administration manager
(D) Bring handouts along to the room

DẠNG 2: MEMO

Questions 6 -7 refer to the following note.

Message taker: Jude Simpson, Receptionist


Message for: Geordan O‘Brien, Operations manager .
Caller: Marianna Gomez, Gomez Repair Center
Caller's contact: 010-552-775 ;
Time: 3:47 p.m., Monday June 18

UNIENGLISHCENTER | 180
Message:

You can collect your vehicle from the repair center this aftemoon. The total cost of the service
comes to $345.50. Acting on your request, the tires have been rotated, and the battery has been
replaced. Also, after the repairmen inspected the rest of the vehicle and realized that what you
said was right, they performed a wheel alignment.

As usual, complimentary oil changes were also provided. You will receive a more detailed
account of the charges and services rendered on pick-up.

6. What was the main purpose of Ms. Gomez's call?


(A) To tell a customer his vehicle is ready for pickup
(B) To inform a client of a mechanical problem with his car
(C) To quote a price estimate for a vehicle check-up
(D) To demand an overdue payment for a repair service

7. What is NOT requested by Mr. O'Brien?


(A) Tire rotation
(B) An oil change
(C) A battery replacement :
(D) Wheel alignment

Questions 8-11 refer to the following memo.

FROM: Vice President of Opt rations


To: All employees

For the past three months, a team from the financial department has been investigating ways for
our company to cut costs. They delivered their final report to me yesterday, and I'd like to share
the results with you.

The number-one way we, here at Symbolite Publishing, can lower our overhead is by conserving
office supplies. For example, the investigative team found that we use over 56,000 sheets of
paper every week. Not only is this excessive, but it is unnecessary. By utilizing digital document
formats and communicating by email, the financial team believes we can reduce that number to
12,000 sheets per week or less. That translates to a yearly saving of nearly $23,000. In order to
encourage smart use of office supplies, I am designing a training workshop to be attended by all
employees. I will meet with department heads tomorrow to determine an appropriate schedule
and format, with the hope of beginning the sessions sometime next month.

Until then, please be conscious of the ways you use office supplies and how you could use less.

UNIENGLISHCENTER | 181
Thank you

8. What is the purpose of the memo?


(A) To announce findings of a financial study :
(B) To explain the delay of a supply delivery
(C) To report the company's sales figures
(D) To thank employees for their performance

9. How much paper does the company currently use per week?
(A) Less than 12,000 sheets
(B) About 12,000 sheets
(C) Nearly 23,000 sheets
(D) More than 56,000 sheets

10. What are employees asked to do?


(A) Try to conserve supplies on their own
(B) Avoid using color printers
(C) Change the office supplies contractor
(D) Organize a training workshop

11. What will happen next month? .


(A) Office supplies will be delivered.
(B) A report will be released.
(C) Workers will receive training.
(D) Department heads will meet.

DANG 3: NOTICE

Questions 12-13 refer to the following notice.

The Global Poverty Fund (GPF) is proud to announce its annual "Save the Children‖ production,
to be held on March 14. The venue for the event is the Gatland Theater,Chicago.

This year‘s production is a performance of John Digby's acclaimed musical The Pilots.
Directed by Kate Mason, and featuring international movie star Francis Wilson as well as
a popular singer Ben Diver in the ead roles, the performance will be fantastic.

As ever, proceeds from the event will go towards alleviating poverty and building educational
programs for children in Africa.

To purchase tickets and support this worthy cause, please contact the Gatland Theater at
900-335-7255. Tickets cost $50 for adults and $35 for children. Group discounts areavailable.

UNIENGLISHCENTER | 182
12. What is the main purpose of the notice?
(A) To publicize a classical concert
(B) To promote a charity fundraiser
(C) To declare the opening of a theater
(D) To announce a change of venue

13. Which of the following is NOT true about the event?


(A) The show features singing and dancing.
(B) Tickets are cheaper for people in groups.
(C) The show stars Kate Mason.
(D) Profits will be donated to charity

DANG 4: ANNOUNCEMENT

Questions 14-16 refer to the following announcement

Attention all Freemont City Literature Society members:

We are happy to announce the upcoming appearance by Allie Garcia at the Freemont City
Auditorium this Thursday at 7:30 p.m. As you know, Ms. Garcia is the award-winning author of
the novel ―Life on the Sea‖. She will be speaking about the art of imaginative fiction writing and
will stay for a cocktail reception afterwards.

As usual, members of the Freemont City Literature Society enjoy half off on tickets to events at
the Freemont City Auditorium. Just remember to bring your membership card with you to the
event. We hope to see you at the auditorium on Thursday evening to take advantage of this
amazing opportunity.

14. Why was this announcement written?


(A) To notify members of a meeting cancellation
(B) To discuss preparations for a cocktail reception
(C) To announce the publication of a book
(D) To inform members of a special event

15. What is taking place at the Freemont City Auditorium?


(A) A literature society presidential election
(B) A talk on creative writing :
(C) A seminar about an author
(D) A private dinner party

16. What will Freemont City Literature Society members receive?

UNIENGLISHCENTER | 183
(A) Discounted admission
(B) Free copies of a novel
(C) New membership cards
(D) The author's autograph

DANG 5: ARTICLE

Questions 17-20 refer to the following article,

Dubai, Jan 14 – JAK Technology announced yesterday that Wilson Ferris has been chosen to
replace Fredrick Tumer as the Senior Director of Finances. His new job responsibilities will
include managing the finance department, developing budget plans, analyzing expenditures, and
projecting future earnings. With more than fifteen years of experience in the finance industry,
Mr. Ferris has a proven record of managing corporate finances and leading large teams of
talented people. According to the president of JAK Technology, Bob Goldberg, Mr. Ferris will
undoubtedly add value to the company by strengthening its role as the world‘s top distributor of
accounting software for large corporations.

Languatec Inc., a company renowned for its popular foreign language education software. While
with Languatec Inc., he was responsible for developing a progressive financial plan that saved
the company millions of dollars — boosting its market value by 25 percent. Following a
successful five years at Languatec, Ferris left to start his own business, Smart Consultants Inc.,
where he offered savvy financial advice to numerous high profile corporations, such as
Blooming Investments.

17. What is Wilson Ferris best known for?


(A) His work as a technology specialist
(B) His background in business finances
(C) The foreign language software he developed
(D) The project he did for Bob Goldberg

18. What kind of service does JAK Technology provide?


(A) Accounting software
(B) Computer repairs
(C) Advertising campaigns
({D) Financial analysis

19. What is mentioned about Ferris’ work at Languatec?


(A) He broke the record for number of sales.
(B) He decided to leave the position after two years.
(C) He increased the company's value.
(D) He introduced a more efficient production process.

UNIENGLISHCENTER | 184
20. Which company was established by Mr. Ferris?
(A) JAK Technology
(B) Languatec Inc.
(C) Smart Consultants Inc.
(D) Blooming Investments

DANG 6: ADVERTISEMENT

Questions 21-22 refer to the following advertisement.

Urban Communications

Satisfying you and your customer's communication needs

At Urban Communications, we know how important it is to communicate quickly and


effectively with customers. That's why we provide a reliable calling service to help you stay in
touch with customers without spending a fortune. Our professional staff can assist your
customers with almost any problem, whether it is answering questions about your company's
products, processing orders, or scheduling appointments. Urban Communications has over 15
years of experience and has worked with some of the largest and most successful corporations in
the country. To stay ahead of our competitors, we continually upgrade our equipment, which
also means that we can consistently offer a high standard of service that meets all of our
customers‘ particular needs. To help your business succeed tomorrow, call us today at 854-623-
3154, or visit website at www urbancommunications.com.

21. What service does Urban Communications offer?


(A) Upgrading computer software
(B) Answering telephones
(C) Repairing electronic equipment
(D) Designing websites

22. What is NOT mentioned about Urban Communications?


(A) It has worked with large companies.
(B) It uses the most recent technology.
(C) It offers weekend service.
(D) It has many years of experience

Questions 23-25 refer to the following advertisement.

Synergy Online Banking

UNIENGLISHCENTER | 185
Access your Synergy Bank account from anywhere in the world!

Synergy Bank is proud to introduce its latest service: Internet Banking. Access your account 24
hours a day, seven days a week, wherever you have an internet connection!
Here are just a few of the options available to you with Synergy Online Banking:

- Log into your account and view current balances in your savings or checking account |

- View an easy-to-read, printable version of all your account transactions

- Quickly transfer money from one account to another

- Make payments online for all your bills

- Update your name and address by completing the form online

Save even MORE time by signing up for our exclusive Synergy Home Banking service
($5.95/month). With this service, you'll be able to easily:

- Ask to receive Synergy Bank's email newsletter, which outlines new products and services,
and offers the latest financial advice

- Have unlimited access to our Financial Resources section, which has all the information
you need to choose the right financial plan for you and your loved ones, and an online calculator
to plan monthly budgets and calculate upcoming bills

To make banking easy, set up your Synergy Bank online services today!
If you currently hold a Synergy Bank account, then just visit our website at
www.synergybank.com and click on the ‗Online Banking Set-Up‘.

23. What is NOT mentioned as an advantage of Synergy Online Banking?


(A) Check out current account balances
(B) Receive e-mail notifications for transactions .
(C) Transfer money from one account to another.
(D) Review history of account activity

24. How can customers notify the bank about changes to personal information?
(A) Fill out the online form
(B) Send an email to customer services
(C) Visit the closest Synergy Bank branch in person
{D) Cail Synergy Bank

UNIENGLISHCENTER | 186
25. What does this advertisement emphasize about the services?
(A) Option to apply for a joan online
(B) Ability to speak with a service agent 24 hours a day
(C) Effortless banking
(D) Fewer fees for banking services

DANG 7: INFORMATION

Questions 26-28 refer to the following document.

Attn.: All employees, Austin Headquarters


Date: April 2, 2008

Here is the in-house schedule for the National Conference in April. The five affiliates will be in
Austin for this year's conference, which means about 200 people will be attending.
April 11 Location
10:00 -11:30 Opening Remarks by CEO Mason Lecture Room 4th floor
11:30 – 1:30 Lunch Dining Room
1:30 – 6:00 Topic: Product Summary & Showroom
Performance
6:00 – 8:00 Dinner TBA

April 12 Location
9:30 -11:30 Tea Innovations Activities Boardroom
11:30 – 1:30 Lunch Dining Room
1:30 – 6:00 Topic: Research & Development Auditorium
6:00 – 8:00 Dinner TBA

Because of the large number of people this year, we will hold the ‗Team Innovations‘ part of the
conference in small meeting rooms. We will divide employees into six groups to be announced
at the end of lunch on April 11. The groups will be chosen with the aim of getting employees
from the different company offices to mix together. If there is anything you would like to
comment on, please email me or come see me in HA, room 202, before April 4.
Laurie Pitt
ipitt@newdesigns.com

26. What kind of event will take place?


(A) A scientific symposium
(B) A company-wide annual meeting
(C) A regional spring conference

UNIENGLISHCENTER | 187
(D) A seminar on fiscal responsibility

27. By what date should changes to the schedule be suggested?


(A) April 2
(B) April 4
(C) April 14
(D) Aprit 12

28. What will determine the organization of 'Team Innovations'?


(A) Employees‘ place of work
(B) The preferences of the affiliate companies
(C) The number of people who attend
(D) How many meeting rooms are available

Questions 29-32 refer to the following form.

Ad World Inc.
Employee Appraisal Form
Employee: Dick Brampton
Position: Marketing Assistant
Date: July 16, 2008
Appraised by: Robin Nagel, Personnel Director .

Please rate the employee according to the following criteria:

Excellent Above Average Average Below Average

Attitude, Motivation x
Initiative x
Efficiency x
Teamwork x
Organizational x
Skills
Punctuality x
Client Relations xx
Corporate Pride x

Other Observations:

- Because Mr. Brampton was behind schedule, his colleague Stephanie Ramirez has
had to finish and submit his last 3 ad proposals.

UNIENGLISHCENTER | 188
- Mr. Brampton spoke with a rude tone to our client at Vistrim on May 14.
- Mr. Brampton received an official warning on June 3 for being consistently late to work.
The warning was issued by Philippe Buldakov, the Marketing Team Supervisor.
Overall Comments:
- Mr. Brampton lacks the professionalism necessary to survive in the advertising industry.
I think we should give Mr. Brampton one last chance to prove himself over a month trial period.
Mr. Buldakov and will meet with him regularly during this time to discuss ways to enhance his
performance. If he still doesn't show any signs of improvement, I strongly recommend that his
employment at Ad World Inc. is terminated.

29. Who most tikely is this form intended for?


(A) Personnel Director
(B) Vistrim's senior executives
(C) The marketing team
(D) Ad World Inc.'s management

30. What is Stephanie Ramirez's role at the company?


(A) Personnel director
(B) The marketing team supervisor
(C) Marketing assistant
(D) Client relations consultant

31. What is Mr. Brampton NOT guilty of doing?


(A) Speaking rudely to his colleagues
(B) Neglecting his duties on projects
(C) Treating a client unprofessionally
(0) Failing to arrive at work on time

32. What does Mr. Nagel suggest?

(A) Assigning Mr. Brampton to a new team


(B) Terminating Mr. Brampton's employment
(C) Putting Mr. Brampton on probation
(D) Issuing Mr. Brampton an official warning

Questions 33-35 refer to the following information.

PeaceValley
Resort Guest Information
Introduction
Thank you for choosing Peace Valley Resort, the only hotel in the Bay Area that caters
exclusively to international business executives. Our reception desk is staffed with individuals

UNIENGLISHCENTER | 189
who speak French, Spanish, German, Japanese and Mandarin. Every room comes with a
wireless Internet connection, and we offer a reduced rate for international phone calls. There is
also a conference room, and a computer lab with computers, fax machines and copying
machines.
To make the most of all our facilities and services, we recommend that you read this pamphlet
thoroughly. For more information on local restaurants, entertainment venues and museums,
consult the ‗Guide to Bay Area Life‘, which is located in the drawer of your night stand. We
sincerely hope that you enjoy your stay; if you need any help, or would like to make a
reservation for meals or tours, please contact Reception at extension 002. :
Table of Contents
Page 1 — Welcome Message from the Manager of Peace Valley Resort
Page 2 — Map of Resort's Facilities ;
Page 4 — List of Hotel Regulations :
Page 6 — Outline of Peace Valley Resort Services
Page 10— Menu for the Peace Valley Resort Gourmet Cafe

33. What service is NOT discussed in the information?


(A) Multi-lingual employees
(B) Internet connection in rooms .
(C) Conference area
(D) Free international calls

34. Where can people find out more about restaurants in the area?
(A) In the guest information pamphlet
(B) In the guide for business travelers
(C) In the local newspaper
(D) In the Guide to Bay Area Life :

35. What information is provided on pages 2-3 of the pamphlet?


(A) A message from the resort manager
(B) A menu for the resort restaurant
(C) The location of the resort fitness center ,
(D) The list of resort regulations

Questions 36 - 38 refer to the following form.


Trenton
Take a moment to tell us how we did. Complete this form and hand it to the cashier for a $5 gift
certificate good at Trenton.
Date: 03/22/08 Time: 6:50 Name of Server: Jeanette
Is this your first time at Trenton?
v Yes x No
Please rate the following (5 = Excellent, 1 = Poor)

UNIENGLISHCENTER | 190
Food Quality
x1 x2 x3 x4 x5
Service
x1 x2 x3 x4 x5
Atmosphere
x1 x2 x3 x4 x5

Additional Comments
The food was good and the service was excellent but | think the atmosphere in the restaurant
could be better. I don't like the decorations on the walls and the carpet on the floor is dirty.

Thank you for helping us better serve you!

36. Where would this form most likely be seen?


(A) In a cookbook
(B) In a restaurant
(C) In a food magazine .
(D) In a gift shop

37. What do customers receive for completing the form?


(A) A coupon
(B) A free meal
(C) A beverage
(D) A food sample

38. What suggestion did the customer make?


(A) The pattern of the carpet should be changed.
(B) The walls should be painted a different color.
(C) The service should be improved.
(D) The interior should be decorated differently.

DANG 8: MIX

Questions 39 - 43 refer to the following email and advertisement.

From: Matthew Remington


To: Tri-State Realty
Subject: House Search
To whom it may concern:

Lam currently living in the Pittsburg area, but will be relocatirig to California, and would like

UNIENGLISHCENTER | 191
some more information on the apartments your agency is currently leasing in the Berkley area.
Although my primary concern is which neighborhood the building is in, my other requirements
are that it is one room, with large windows, and a good view.

I would really like to start the process of searching for an apartment as soon as possible because
I have accepted a teaching position at Berkley University, and will be starting work in less than
two months. Also, since I don't own a car, I would prefer to live within walking distance of the
University campus. I certainly appreciate your assistance with this matter. Please feel free to
contact me any time at 345-243-5675.

Regards,
Matthew Remington

Tri-State Realty - Find a home without a hassle


Here are just a few of the properties we offer:

Bayview Apartments
This lovely complex is located in the center of the city, just a few blocks away from all the
hottest shopping mails and restaurants. All apartments have two rooms and come fully
furnished. Utilities and internet access are included in the monthly rent.

Covent Gardens
Located right beside the Berkley University campus, this beautiful complex is perfect for
students looking to escape school life. Choose from one, two or three room apartments.
The complex also includes a swimming pool, fitness center and tennis courts.

Royal Suites
For anyone in search of superior living facilities, look no further than Royal Suites. These
fine apartments have two bedrooms, a living room, a dining room, two bathrooms, a sun
porch and an entertainment room. Special service features include valet parking, a
gourmet restaurant, fitness center and spa.

Goldfinch Forest
Families are welcome at Goldfinch Forest, a new housing complex providing economical
but comfortable housing away from the noise of the city. All apartments come with a
washing machine and dryer, oven and refrigerator. Plus, there is an onsite daycare
facility and playground.

For more information about our properties, or to schedule an appointment to meet with
one of our real estate agents just call 1-888-956-4357. You can also email us at
inquiry@tristaterealty.com, or visit our company website.

UNIENGLISHCENTER | 192
39. What is Mr. Remington's first concern?
(A) The location of the apartment
(B) The cost of monthly utilities
(C) The size of the rooms
(D) The fumishing provided

40. Why is Mr. Remington planning to relocate?


(A) He is about to graduate from college. :
(B) He is going to begin a new job.
(C) He is unhappy with his current apartment.
(D) He is tired of living outside the city. .

41. Which apartment building would most likely interest Mr-Remington?


(A) Royal Suites
(B) Bayview Apartments ,
(C) CoventGardens
(D) GoldfinchForest

42. What does the advertisement suggest about the Goldfinch Forest?
(A) It is close to popular city attractions.
(B) It is an exclusive housing complex.
(C) it is excellent for University students.
(D) It is in a quiet neighborhood.

43. Which is NOT mentioned as a way to contact Tri-State Reality?


(A) Telephone
(B) Email
(C) Fax
(D) Internet .

Questions 44 - 48 refer to the following two emails.

From: Rey Macintosh, Jennings Hotel


To: Barbara Elk
Subject: Satisfaction Rating
Dear Mrs. Elk,

Thank you for your recent patronage at the Jennings Hotel. I am writing this email to you to
kindly ask your opinion on how you enjoyed your stay at the Jennings Hotel so that we can
ensure that we continue to offer only the best services and facilities.
We pride ourselves on offering exceptional service, outstanding facilities and affordable prices.

UNIENGLISHCENTER | 193
Our unique atmosphere and friendly service sets us apart from other hotels, and brings a new
standard of excellence to the hotel industry. With over 250 rooms, three restaurants, a golf
course and live entertainment on Friday and Saturday nights, the Jennings Hotel is the perfect
place to relax. In addition to the services already mentioned, we are also the only hotel in the
area to offer a comprehensive fitness center, which is located on the second floor. Guests can
choose from over 150 different types of exercise equipment, including tread mills, stationary
bikes, stair-steppers and weight machines.
Your response is greatly appreciated, and I assure you that any information you provide,
including your name, address and occupation, will remain anonymous.
Sincerely
Roy Macintosh

Director of Customer Relations


Jennings Hotel
From: Barbara Elk
To: Roy Macintosh
Subject: Re: Satisfaction Rating
Dear Mr. Macintosh,
I received your email regarding my recent stay at the Jennings Hotel. It is nice to see that a hotel
cares so much about its guests! To be honest, in general, my stay at the hotel was exceptional.
The staff was very friendly, my room was luxurious and the food at the restaurants was
excellent. What I enjoyed most about your hotel, however, was the facility on the second floor. I
was a guest lecturer at the Wellstone Health Conference, and was required to work long hours
preparing and delivering seminars. I was surprised to discover that I had 24-hour access to the
facility, which was very convenient. I was disappointed, though, that the hotel didn't have a
business center. It would have beennice, especially since 1 often had to send documents to
people at the last minute. Aside from this, I was very pleased with my experience at the Jennings
Hotel, and would certainly conic again.
Sincerely,
Barbara Elk

44. What is the purpose of the email from Roy Macintosh? ,


(A) To inquire about hotel reservations
(B) To collect information from the customer
(C) To introduce a new feature of the hotel :
(D) To explain the hote!'s rules and regulations

45. What is NOT a service offered at the hotel?


(A) Golf course
(B) Fitness center ‗
(C) Airport shuttle
(D) Weekend entertainment

UNIENGLISHCENTER | 194
46. What does Mr. Macintosh tell Mrs. Elk?
(A) The bill for her stay will be charged to her credit card.
(B) The hotel is offering a special rate.
(C) The room she requested is not available.
(D) The information she provides will be kept confidential.

47. What does Mrs. Elk mention she liked most about the hotel?
{A) The exceptional fitness facilities
{B) The show offered on Saturday night
(C) The business center and computer lab
(D) The service at the first floor restaurant

48. What is probably true about Mrs. Eik?


(A) She wanted to extend her stay.
{B) She will stay at the hotel again.
{C) She will speak at the conference next year.
(D) She didn‘t expect the staff to be so friendly.

Questions 49 - 53 refer to the following notice and email.

Attention shoppers!

Following a routine engineering safety inspection, experts recommended. that we have the
ceiling in the Kindle Street parking lot reinforced. As a result, this lot will be closed for
renovation from the 13th to the 15th of this month. It will reopen again on the 16th. We
apologize for any inconvenience this causes.

On the affected days you will need to use either our main parking lot or the nearby Nelson
public lot. For those of you who don‘t know, Nelson Street is on the other side of McLean Park,
and the public lot is next to Kennedy Sports Center. We have made arrangements with the
management at Nelson that will allow us to validate tickets for that lot.
Thank you,
The Management, Luton Department Store

From: Max Verlaine <maxyaho@sendyou.net>


To: Customer Affairs, Luton Department Store <caservice@lutonstore.com>
Subject: Parking

Hello,

UNIENGLISHCENTER | 195
I visited your department store this morningto do some shopping. Your main parking area was
full, and your Kindle Street lot was under maintenance, so I had to park at the public lot on
Nelson Street. This was a little inconvenient as the walk from Nelson Street is uphill. However,
I understand the situation and the safety concerns involved.

I was very disappointed, though, when I had so much trouble getting my parking ticket validated
at your store. The clerk who served me claimed that she was not authorized to handle tickets for
the Nelson public lot. She told me I had to go to the information desk. Then, in tum, the person
at the information desk directed me to the cashier's office on the third floor. I finally got the
ticket validated there. The process was very confusing and complicated. | realize that today is the
first day your employees have had to handle parking validation for the Nelson lot, but please
make sure your staff is fully aware of your procedures in the future.
Yours,
Max Verlaine

49. What is the notice mainly about?


(A) A temporary closure of a parking area 51. When did Mr. Verlaine go shopping?
(B) The preparations for a routine safety (A) The 13th
inspection (B) The 14th
(C) Proposed changes to the layout of the (C) The 15th
department store (D) The 16th
(D) Some upcoming road construction work
52. Why has Mr. Verlaine written this
50. What caused the management at email? .
Luton to take action? (A) To complain about a problem he
(A) A serious disaster experienced at the store
(B) A building check (B) To ask for a department store
(C) New engineering regulations membership card
(D) Relocation of a parking lot (C) To request a parking permit for the main
lot
(D) To inquire about other parking facilities

53. Where are tickets for the alternative


parking lot validated?
(A) At the office on Nelson Street
(B) At the cashier's office
(C) At the counter
(D) At the information desk

UNIENGLISHCENTER | 196

You might also like